Напряжение по закону ома: Электрическое сопротивление. Закон Ома для участка электрической цепи — урок. Физика, 8 класс.

Содержание

формула, определение простыми словами, задачи с решением

Закон Ома — главный закон электротехники, который открыл в 1826 году выдающийся немецкий ученый Георг Симон Ом. Вместе с экспертом разберем формулировку, формулу и задачи на закон Ома с решением

Борис Михеев

Автор КП

Николай Герасимов

Старший преподаватель физики в
Домашней школе «ИнтернетУрок»

Физика — наука эмпирическая. Ее основные законы вытекают из практического опыта и частенько много лет не имеют теоретических обоснований. Именно так обстоит дело с главным законом электротехники, который открыл в 1826 году выдающийся немецкий ученый Георг Симон Ом.

Электрические явления люди наблюдали сотни лет. Но никак не связывали между собой заряженность потертого янтаря и молнию. Только на исходе XVIII столетия электричество стали внимательно исследовать. В 1795 году Алессандро Вольта изобрел «вольтов столб», химическую батарею, и обнаружил появление тока в проводнике, соединяющем ее полюса. Сферы применения электричества стремительно множились, и появилась острая необходимость в расчетных формулах для инженеров. Эту задачу решали многие ученые, но первым сформулировал главную формулу электротехники именно Георг Ом. Он ввел в обиход понятие сопротивления и опытным путем установил зависимость между основными характеристиками электрической цепи.

Определение закона Ома простыми словами

Электрическая цепь состоит из двухполюсного источника напряжения, то есть батареи, аккумулятора или генератора. Если полюса источника соединить проводами, то по ним потечет электрический ток. Его величина определяется сопротивлением проводников. Наглядное представление этой зависимости — обыкновенный водопровод. Аналогом источника напряжения является насос или водонапорная башня, создающая давление в магистрали, количество воды, прошедшее по трубе, — подобие силы тока, а кран соответствует сопротивлению. Полностью открытый, он не ограничивает поток, по мере закручивания отверстие для воды уменьшается, пока не закроется совсем.

Закон Ома для участка цепи

Опытным путем исследователь установил взаимосвязь характеристик электрической цепи. Классическая формулировка закона Ома звучит так:

«Сила тока на участке цепи прямо пропорциональна напряжению и обратно пропорциональна сопротивлению».

Формула закона Ома для участка цепи

Где I – сила тока, измеряемая в Амперах (А), U – напряжение, измеренное в Вольтах (В), R – сопротивление, измеряемое в Омах (Ом).

В таком виде закон Ома приведен в школьных учебниках физики. Согласно этой простой формуле, для определения уровня тока в проводнике достаточно величину напряжения на его сторонах разделить на некий условно постоянный коэффициент, то есть на сопротивление. Почему «условно»? Потому что величина сопротивления может меняться в зависимости от температуры. Поэтому, кстати, лампы накаливания чаще всего перегорают при включении. Сопротивление холодной спирали ниже, чем нагретой, скачок тока при подаче напряжения вызывает ее резкое расширение и разрыв. Но если этот момент преодолен и нить накала уцелела, то ее сопротивление растет, и ток ограничивается. А при температуре жидкого гелия, например, сопротивление падает до нуля, наступает сверхпроводимость.

Закон Ома для замкнутой полной цепи

Предыдущая формулировка годится только для участка цепи, где отсутствует сам источник электродвижущей силы. В реальности ток течет по замкнутому контуру, где обязательно есть батарея или генератор, имеющий собственное внутреннее сопротивление. Поэтому формула закона Ома для полной цепи выглядит несколько сложнее

    Формула закона Ома для замкнутой полной цепи

    Где I – сила тока, измеряемая в Амперах (А), Е – электродвижущая сила, измеренная в Вольтах (В), R – сопротивление, измеряемое в Омах (Ом), r — внутреннее сопротивление источника ЭДС.

    Применение закона Ома

    Георг Ом дал в руки инженеров средство для решения задач, связанных с электрическими цепями. Тепловые и световые приборы, электродвигатели, генераторы, линии электропередач, кабели связи рассчитываются на основе этой простой формулы. Нет такой области электротехники, где она не находит применения. Даже в радиотехнике используется закон Ома, но в дифференциальной форме. «Все гениальное — просто», как считали Еврипид, Леонардо да Винчи, Наполеон Бонапарт и Альберт Эйнштейн, несомненные гении. Закон Ома целиком и полностью подтверждает эту истину.

      Задача на закон Ома с решением

      Задача для участка электрической цепи

      Электрочайник, включенный в сеть с напряжением 220 В, потребляет ток 1,1 А. Каково сопротивление электрочайника.

      Дано:
      U = 220 В
      I = 1,1 А

      Решение:
      Согласно закону Ома для участка цепи:
      R=U/I=220/1,1=200 Ом

      Ответ: R = 200 Ом.

        Задача для полной замкнутой цепи

        Источник постоянного тока с ЭДС E = 24 В и внутренним сопротивлением r = 1,5 Ом замкнут на внешнее сопротивление R = 11 Ом. Определить силу тока в цепи.

        Дано:
        Е=24 В, r=1,5 Ом, R = 11 Ом

        Решение:
        По закону Ома для замкнутой цепи: I = E/(R + r) = 24/(11+1,5) = 1,92 А.

        Ответ: I=1, 92 А.

          Популярные вопросы и ответы

          Отвечает Николай Герасимов, старший преподаватель физики в Домашней школе «ИнтернетУрок»

          Сколько всего законов Ома в физике?

          — Существует два закона Ома: закон Ома для участка цепи и закон Ома для полной (замкнутой) цепи. Первый связывает сопротивление участка, силу тока в нём и разность потенциалов (напряжение) на его концах. Кроме того, в нем отражено наличие в цепи источника тока.

          Второй учитывает и потребителей электрического тока (электрические лампы, обогреватели, телевизоры и так далее), и его источники (генераторы, батарейки, аккумуляторы). Дело в том, что любой источник тока обладает внутренним сопротивление, которое влияет на силу тока. Именно это и учитывается в законе Ома для полной (замкнутой) цепи.

          При каких условиях выполняется закон Ома?

          Согласно закону Ома, существует линейная зависимость между силой тока в участке цепи и напряжением на его концах. Он отлично выполняется для металлических проводников при любых напряжениях, а вот для тока в вакууме, газе, растворах или расплавах электролитов, полупроводниках линейная зависимость нарушается, и применять закон Ома в том виде, в котором его изучают в школьном курсе, уже нельзя.

          Для чего нужен закон Ома?

          — Трудно переоценить значимость этого закона. Он позволил производить расчет электрических цепей, без которых практически невозможно представить жизнь современного человека, так как они лежат в основе любого электроприбора, начиная от обычной лампы накаливания и заканчивая самыми современными компьютерами.

          В каком классе проходят закон Ома?

          — В школьном курсе ученики впервые знакомятся с электрическими явлениями и законом Ома для участка цепи в 8 классе. Более подробно о причинах возникновения электрического тока и его источниках ученики знакомятся в курсе старшей школы (10 или 11 класс, в зависимости от программы). Здесь же ученики впервые встречаются и с законом Ома для полной (замкнутой) цепи.

          Фото на обложке: pixabay.com

          Закон Ома для участка цепи | 8 класс

          Содержание

            Для описания процесса протекания электрического тока в цепи у нас есть уже три характеристики: сила тока, напряжение и сопротивление.

            Мы выяснили, что некоторые из них связаны между собой. Сила тока зависит от напряжения. Эти величины прямо пропорциональны друг другу. Во сколько раз увеличивается напряжение на концах проводника, во столько же раз увеличивается сила тока в нем. Проводник мы меняли в этих опытах, сопротивление оставалось постоянным.

            Далее мы узнали, что сила тока зависит и от электрического сопротивления проводника. Показания амперметра при подключении в цепь разных проводников менялись. Напряжение при этом оставалось в этих проводниках постоянным.

            Но мы пока не установили, каким образом между собой связаны сразу эти три величины. На данном уроке мы опытным путем докажем эту связь и познакомимся с законом Ома для участка цепи.

            Опытное определение зависимости силы тока от сопротивления при постоянном напряжении

            Для того, чтобы определить зависимость силы тока от сопротивления проводника, мы проведем еще один опыт. Теперь мы будем знать электрическое сопротивление тех проводников, которые будем использовать.

            Обратите внимание, что в ходе опыта напряжение на концах используемых проводников должно быть постоянным. Эта величина не должна изменяться, чтобы мы могли корректно оценить зависимость силы тока от сопротивления.

            Соберем электрическую цепь из источника тока, ключа, амперметра, проводника. К проводнику параллельно подсоединим вольтметр (рисунок 1).

            Проводников у нас будет три разных. Они обладают разными сопротивлениями. Мы будем поочередно подключать их в цепь. Каждый раз мы будем фиксировать показания амперметра.

            По показаниям вольтметра необходимо следить, чтобы напряжение на концах каждого проводника было одинаковым.

            Рисунок 1. Установление зависимости силы тока от сопротивления проводника
            {"questions":[{"content":"При экспериментальном исследовании зависимости силы тока в проводнике от его сопротивления необходимо следить, чтобы[[choice-1]]","widgets":{"choice-1":{"type":"choice","options":["напряжение на концах проводника оставалось постоянным","вольтметр был подсоединен в цепь последовательно","сила тока в цепи не изменялась"],"explanations":["","Вольтметр НИКОГДА не подсоединяется в цепь последовательно. Только параллельно.","Сила тока в цепи будет равна силе тока в проводнике. Если мы говорим об определении зависимости, это подразумевает, что исследуемые величины будут изменяться."],"answer":[0]}}}]}

            Связь силы тока и сопротивления

            Проведя все измерения, мы занесли их результаты в таблицу 1.

            № опытаНапряжение на концах проводника, $В$Сопротивление проводника, $Ом$Сила тока в цепи, $А$
            1212
            2221
            3240,5
            Таблица 1. Данные, полученные при проведении опытов

            Давайте проанализируем наши результаты.

            В первом опыте сила тока составила $2 \space А$ при сопротивлении проводника в $1 \space Ом$.

            Для следующего опыта мы взяли проводник с сопротивлением в $2 \space Ом$. Это в два раза больше, чем в первом опыте. А вот сила тока составила $1 \space А$. Она стала в два раза меньше.

            В третьем опыте сопротивление проводника было равно $4 \space Ом$. То есть, в четыре раза больше, чем в первом опыте. Сила тока стала равна $0.5 \space А$. Она уменьшилась в четыре раза.

            Напряжение на концах проводников во всех трех опытах оставалось постоянным. Оно было равно $2 \space В$.

            Так какова зависимость силы тока в проводнике от сопротивления этого проводника?

            Из наших опытов мы можем сделать определенный вывод.

            Сила тока в проводнике обратно пропорциональна сопротивлению проводника.

            {"questions":[{"content":"Сопротивление проводника и сила тока в нем[[choice-6]]","widgets":{"choice-6":{"type":"choice","options":["обратно пропорциональны друг другу","прямо пропорциональны друг другу","не зависят друг от друга"],"explanations":["","Так связаны между собой сила тока и напряжение.",""],"answer":[0]}}}]}

            Закон Ома для участка цепи

            В 1827 году немецкий физик Георг Ом (рисунок 2) открыл закон, описывающий зависимость силы тока от напряжения на концах проводника и сопротивления этого проводника. Проводник является частью электрической цепи. Этот закон был назван в честь этого ученого и называется законом Ома для участка цепи.

            Рисунок 2. Георг Симон Ом (1789 — 1854) — немецкий физик. Вывел и подтвердил один из основных физических законов — закон Ома

            Как формулируется закон Ома?

            Сила тока в участке цепи прямо пропорциональна напряжению на концах этого участка и обратно пропорциональна его сопротивлению.
            $I = \frac{U}{R}$,
            где $I$ — сила тока в участке цепи, $U$ — напряжение на концах этого участка, $R$ — электрическое сопротивление участка.

            {"questions":[{"content":"Формула, описывающая закон Ома для участка цепи, имеет следующий вид:[[choice-10]]","widgets":{"choice-10":{"type":"choice","options":["$I = \\frac{U}{R}$","$U = \\frac{I}{R}$","$I = \\frac{q}{t}$","$U = \\frac{A}{q}$"],"explanations":["","","Это определение силы тока.","Это определение напряжения."],"answer":[0]}}}]}

            График зависимости силы тока от сопротивления проводника

            На рисунке 3 изображен график зависимости силы тока от сопротивления. Напряжение при этом на концах проводника остается постоянным.

            Рисунок 3. График зависимости силы тока в цепи от сопротивления

            Здесь по горизонтальной оси отложены сопротивления проводников в омах, а по вертикальной — сила тока в амперах.

            Формулы для вычисления напряжения и сопротивления

            Из формулы $I = \frac{U}{R}$ мы можем выразить напряжение и сопротивление:

            $U = IR$,
            $R = \frac{U}{I}$.

            При расчете сопротивления проводника помните, что $R$ — постоянная величина для каждого проводника. Она не будет изменяться при изменениях силы тока или напряжения.

            {"questions":[{"content":"При увеличении напряжения на участке цепи[[choice-17]]","widgets":{"choice-17":{"type":"choice","options":["сопротивление этого участка не изменится","сопротивление этого участка увеличится","сопротивление этого участка уменьшится"],"explanations":["Прямо пропорционально напряжению увеличится сила тока в цепи. Сопротивление является свойством самого проводника и не зависит от изменений напряжения или силы тока. ","",""],"answer":[0]}}}]}

            Упражнения

            Упражнение №1

            Напряжение на зажимах электрического утюга $220 \space В$, сопротивление нагревательного элемента утюга $50 \space Ом$. Чему равна сила тока в нагревательном элементе?

            Дано:
            $U = 220 \space В$
            $R = 50 \space Ом$

            $I — ?$

            Показать решение и ответ

            Скрыть

            Решение:

            Закон Ома для участка цепи: $I = \frac{U}{R}$.

            Рассчитаем силу тока:
            $I = \frac{220 \space В}{50 \space Ом} = 4.4 \space А$.

            Ответ: $I = 4.4 \space А$.

            Упражнение №2

            Сила тока в спирали электрической лампы $0.7 \space А$, сопротивление лампы $310 \space Ом$. Определите напряжение, под которым находится лампа.

            Дано:
            $I = 0.7 \space А$
            $R = 310 \space Ом$

            $u — ?$

            Показать решение и ответ

            Скрыть

            Решение:

            Закон Ома для участка цепи: $I = \frac{U}{R}$.

            Выразим отсюда напряжение и рассчитаем его:
            $U = IR$,
            $U = 0. 7 \space А \cdot 310 \space Ом = 217 \space В$.

            Ответ: $U = 217 \space В$.

            Упражнение №3

            Каким сопротивлением обладает вольтметр, рассчитанный на $150 \space В$, если сила тока в нем не должна превышать $0.01 \space А$?

            Дано:
            $U_{max} = 150 \space В$
            $I_{max} = 0.01 \space А$

            $R — ?$

            Показать решение и ответ

            Скрыть

            Решение:

            Запишем закон Ома для участка цепи: $I = \frac{U}{R}$.

            Выразим отсюда сопротивление и рассчитаем его значение, используя максимальные значения напряжения и силы тока, соответствующие прибору:
            $R = \frac{U_{max}}{I_{max}}$,
            $R = \frac{150 \space В}{0.01 \space А} = 15 \space 000 \space Ом = 15 \space кОм$.

            Ответ: $R = 15 \space кОм$.

            Упражнение №4

            Определите по графику (рисунок 4) сопротивление проводника.

            Рисунок 4. График зависимости силы тока в проводнике от напряжения

            Возьмем из графика данные. При напряжении, равном $10 \space В$, сила тока в проводнике равна $2. 5 \space А$. Запишем условие задачи и решим ее.

            Обратите внимание, что сопротивление $R$ не зависит ни от силы тока, ни от напряжения. Поэтому вы можете выбирать другие значения силы тока и напряжения из графика. Ваш ответ к этой задаче от этого не изменится.

            Дано:
            $U = 10 \space В$
            $I = 2.5 \space А$

            $R — ?$

            Показать решение и ответ

            Скрыть

            Решение:

            Запишем закон Ома для участка цепи: $I = \frac{U}{R}$.

            Выразим отсюда сопротивление и рассчитаем его значение, используя данные графика:
            $R = \frac{U}{I}$,
            $R = \frac{10 \space В}{2.5 \space А} = 4 \space Ом$.

            Ответ: $R = 4 \space Ом$.

            Упражнение №5

            Рассмотрите рисунок 1 и таблицу результатов опыта, выполняемого в соответствии с этим рисунком. Что изменится на рисунке и в схеме электрической цепи, когда будут проводиться опыты №2 и №3, указанные в таблице 1?

            Для опыта №2:
            на рисунке будет подключен другой проводник, имеющий сопротивление $2 \space Ом$. Амперметр будет показывать силу тока, равную $1 \space A$.

            Для опыта №3:
            на рисунке ничего не изменится. Это иллюстрация именно этого опыта (в цепь подключен проводник с сопротивлением $4 \space Ом$).

            Схема электрической цепи будет одинаковой для всех трех опытов, если не отмечать сопротивление проводника (рисунок 5).

            Рисунок 5. Схема электрической цепи для опытов по определения зависимости силы тока от сопротивления

            Упражнение №6

            По показаниям приборов (рисунок 6) определите сопротивление проводника АВ.

            Рисунок 6. Зависимость силы тока от свойств проводников

            Дано:
            $U = 4 \space В$
            $I = 1 \space А$

            $R — ?$

            Показать решение и ответ

            Скрыть

            Решение:

            Выразим сопротивление из закона Ома для участка цепи:
            $I = \frac{U}{R}$,
            $R = \frac{U}{I}$.

            Из прошлого урока: этот проводник AB — железная проволока. Рассчитаем ее сопротивление:
            $R = \frac{4 \space В}{1 \space А} = 4 \space Ом$.

            Ответ: $R = 4 \space Ом$.

            Упражнение №7

            На рисунке 7 изображены графики зависимости силы тока от напряжения для двух проводников А и В. Какой из этих проводников обладает большим сопротивлением? Определите сопротивление каждого из проводников.

            Рисунок 7. Графики зависимости силы тока от напряжения для двух проводников

            Возьмем данные из графиков. Для проводника A выберем напряжение, равное $6 \space В$. При таком напряжении сила тока в этом проводнике будет равна $3 \space А$. Для проводника B возьмем значение напряжения, равное $4 \space В$. Ему соответствует сила тока, равная $1 \space А$. Теперь мы можем записать условия задачи и решить ее.

            Дано:

            $U_A = 6 \space В$
            $I_A = 3 \space А$
            $U_B = 4 \space В$
            $I_B = 1 \space А$

            $R_A — ?$
            $R_B — ?$

            Показать решение и ответ

            Скрыть

            Решение:

            Закон Ома для участка цепи: $I = \frac{U}{R}$.
            Выразим сопротивление: $R = \frac{U}{I}$.

            Рассчитаем сопротивление для проводника A:
            $R_A = \frac{U_A}{I_A}$,
            $R_A = \frac{6 \space В}{3 \space А} = 2 \space Ом$.

            Рассчитаем сопротивление для проводника B:
            $R_B = \frac{U_B}{I_B}$,
            $R_B = \frac{4 \space В}{1 \space А} = 4 \space Ом$.

            Проводник B обладает большим сопротивлением, чем проводник A. Его сопротивление больше в 2 раза $(\frac{R_B}{R_A} = \frac{4 \space Ом}{2 \space Ом} = 2$).

            Ответ: сопротивление проводника B в 2 раза больше сопротивления проводника A; $R_A = 2 \space Ом$, $R_B = 4 \space Ом$.

            простыми словами с примерами для “чайников”

            Содержание

            Закон Ома для участка цепи

            Закон Ома для участка цепи гласит, что сила тока (I) на участке электрической цепи прямо пропорциональна напряжению (U) на концах участка цепи и обратно пропорциональна его сопротивлению (R).

            Закон Ома для замкнутой цепи

            Если к источнику питания подключить внешнюю цепь сопротивлением R, в цепи пойдёт ток с учётом внутреннего сопротивления источника:

            I — Сила тока в цепи. Электродвижущая сила (ЭДС) — величина напряжения источника питания не зависящая от внешней цепи (без нагрузки). Характеризуется потенциальной энергией источника.

            r — Внутреннее сопротивление источника питания. Для электродвижущей силы внешнее сопротивление R и внутреннее r соединены последовательно, значит величина тока в цепи определится значением ЭДС и суммой сопротивлений: I =/(R+r) .

            Напряжение на выводах внешней цепи определится исходя из силы тока и сопротивления R соотношением, которое уже рассматривалось выше: U = IR.

            Напряжение U, при подключении нагрузки R, всегда будет меньше чем ЭДС на величину произведения I*r, которую называют падением напряжения на внутреннем сопротивлении источника питания. С этим явлением мы сталкиваемся достаточно часто, когда видим в работе частично разряженные батарейки или аккумуляторы. По мере разряда, увеличивается их внутреннее сопротивление, следовательно, увеличивается падение напряжение внутри источника, значит уменьшается внешнее напряжение U = — I*r. Чем меньше ток и внутреннее сопротивление источника, тем ближе по значению его ЭДС и напряжение на его выводах U.

            Если ток в цепи равен нулю, следовательно, = U. Цепь разомкнута, ЭДС источника равна напряжению на его выводах. В случаях, когда внутренним сопротивлением источника можно пренебречь (r ≈ 0), напряжение на выводах источника будет равно ЭДС (≈ U ) независимо от сопротивления внешней цепи R. Такой источник питания называют источником напряжения.

            Закон Ома для переменного тока

            При наличии индуктивности или ёмкости в цепи переменного тока необходимо учитывать их реактивное сопротивление. В таком случае запись Закона Ома будет иметь вид: I = U/Z

            Здесь Z — полное (комплексное) сопротивление цепи — импеданс. В него входит активная R и реактивная X составляющие. Реактивное сопротивление зависит от номиналов реактивных элементов, от частоты и формы тока в цепи. Более подробно ознакомится с комплексным сопротивлением можно на страничке импеданс.

            С учётом сдвига фаз φ, созданного реактивными элементами, для синусоидального переменного тока обычно записывают Закон Ома в комплексной форме.

            Нелинейные элементы и цепи

            Закон Ома не является фундаментальным законом природы и может быть применим в ограниченных случаях, например, для большинства проводников. Его невозможно использовать для расчёта напряжения и тока в полупроводниковых или электровакуумных приборах, где эта зависимость не является пропорциональной и её можно определять только с помощью вольтамперной характеристики (ВАХ).

            К данной категории элементов относятся все полупроводниковые приборы (диоды, транзисторы, стабилитроны, тиристоры, варикапы и т.д.) и электронные лампы. Такие элементы и цепи, в которых они используются, называют нелинейными.

            Определение единицы сопротивления — Ом

            1 Ом представляет собой электрическое сопротивление участка проводника, по которому при напряжении 1(Вольт) протекает ток 1(Ампер).

            Напряжение, ток и сопротивление

            Электрическая цепь образуется, когда создается проводящий путь, позволяющий электрическому заряду непрерывно перемещаться. Это непрерывное движение электрического заряда по проводникам цепи называется током, и о нем часто говорят как о «потоке», как о потоке жидкости через полую трубу.

            Сила, побуждающая носители заряда «течь» по цепи, называется напряжением. Напряжение – это особая мера потенциальной энергии, которая всегда относительна между двумя точками. Когда мы говорим об определенной величине напряжения, присутствующего в цепи, мы имеем в виду измерение потенциальной энергии для перемещения носителей заряда из одной конкретной точки этой цепи в другую конкретную точку. Без упоминания двух конкретных точек термин «напряжение» не имеет значения.

            Ток, как правило, проходит через проводники с некоторой степенью трения или противодействия движению. Это противодействие движению правильнее называть сопротивлением. Величина тока в цепи зависит от величины напряжения и величины сопротивления в цепи, препятствующего прохождению тока. Как и напряжение, сопротивление – это величина, измеряемая между двумя точками. По этой причине величины напряжения и сопротивления часто указываются как «между» двумя точками в цепи.

            Единицы измерения: вольт, ампер и ом

            Чтобы иметь возможность делать осмысленные утверждения об этих величинах в цепях, нам нужно уметь описывать их количества так же, как мы могли бы количественно определить массу, температуру, объем, длину или любые другие физические величины. Для массы мы можем использовать единицы «килограмм» или «грамм». Для температуры мы можем использовать градусы Фаренгейта или градусы Цельсия. В таблице ниже приведены стандартные единицы измерения электрического тока, напряжения и сопротивления:

            Единицы измерения тока, напряжения, сопротивления в таблице:

            ВеличинаСимволЕдиница измеренияСокращение единицы измерения
            ТокIАмперА
            НапряжениеVВольтВ
            СопротивлениеRОмОм

            «Символ», присвоенный каждой величине, представляет собой стандартную букву латинского алфавита, используемую для представления этой величины в формулах. Подобные стандартизированные буквы распространены во всех физических и технических дисциплинах и признаны во всем мире. «Сокращение единицы измерения» для каждой величины представляет собой алфавитный символ(ы), используемый в качестве сокращенного обозначения конкретной единицы измерения.

            Каждая единица измерения названа в честь известного экспериментатора в области электричества: ампер в честь француза Андре М. Ампера, вольт в честь итальянца Алессандро Вольта, а ом в честь немца Георга Симона Ома.

            Математический символ для каждой величины также имеет значение. «R» для сопротивления и «V» для напряжения говорят сами за себя («Resistance» и «Voltage», соответственно), тогда как «I» для тока кажется немного странным. Предполагается, что буква «I» должна представлять «интенсивность» («Intensity»)(потока заряда). Судя по исследованиям, которые мне удалось провести, кажется, что есть некоторые разногласия по поводу значения слова «I». Другой символ напряжения, «E», означает «электродвижущую силу» («Electromotive force»). Символы «E» и «V» по большей части взаимозаменяемы, хотя в некоторых текстах «E» зарезервировано для обозначения напряжения на источнике (таком как батарея или генератор), а «V»– для обозначения напряжения на любом другом элементе.

            Все эти символы выражаются заглавными буквами, за исключением случаев, когда величина (особенно напряжение или ток) описывается в терминах короткого периода времени (так называемые «мгновенные» значения). Например, напряжение батареи, которое стабильно в течение длительного периода времени, будет обозначаться заглавной буквой «E», тогда как пиковое напряжения при ударе молнии в тот самый момент, когда она попадает в линию электропередачи, скорее всего, будет обозначаться строчной буквой «е» (или строчной буквой «v»), чтобы отметить это значение как имеющееся в один момент времени. Это же соглашение о нижнем регистре справедливо и для тока: строчная буква «i» представляет ток в некоторый момент времени. Однако большинство измерений в цепях постоянного тока, которые стабильны во времени, будут обозначаться заглавными буквами.

            Кулон и электрический заряд

            Одна из основных единиц электрических измерений, которую часто преподают в начале курсов электроники, но нечасто используют впоследствии, – это кулон – единица измерения электрического заряда, пропорциональная количеству электронов в несбалансированном состоянии. Один кулон заряда соответствует 6 250 000 000 000 000 000 электронов. Символом количества электрического заряда является заглавная буква «Q», а единица измерения кулонов обозначается «Кл». Единица измерения тока, ампер, равна 1 кулону заряда, проходящему через заданную точку в цепи за 1 секунду. В этом смысле, ток – это скорость движения электрического заряда через проводник.

            Как указывалось ранее, напряжение – это мера потенциальной энергии на единицу заряда, доступная для стимулирования протекания тока из одной точки в другую. Прежде чем мы сможем точно определить, что такое «вольт», мы должны понять, как измерить эту величину, которую мы называем «потенциальной энергией».

            Общей метрической единицей измерения энергии любого вида является джоуль, равный количеству работы, совершаемой силой в 1 ньютон при движении на 1 метр (в том же направлении). В этих научных терминах 1 вольт равен 1 джоулю электрической потенциальной энергии на (деленному на) 1 кулон заряда. Таким образом, 9-вольтовая батарея выделяет 9 джоулей энергии на каждый кулон заряда, проходящего через цепь.

            Эти единицы и символы электрических величин станут очень важны, когда мы начнем исследовать отношения между ними в цепях.

            Формула закона Ома

            Основное открытие Ома заключалось в том, что величина электрического тока, протекающего через металлический проводник в цепи, при любой заданной температуре прямо пропорциональна напряжению, приложенному к нему. Ом выразил свое открытие в виде простого уравнения, описывающего взаимосвязь напряжения, тока и сопротивления:

            [E=IR]

            В этом алгебраическом выражении напряжение (E) равно току (I), умноженному на сопротивление (R). Используя алгебру, мы можем преобразовать это уравнение в других два варианта, решая его для I и R соответственно:

            [I = frac{E}{R}]

            [R = frac{E}{I}]

            Сила тока, Закон Ома, формула.

            Сила тока в проводнике прямо пропорциональна напряжению и обратно пропорциональна сопротивлению.

            [I=frac{U}{R}]

            Сила тока, Закон Ома, формула.

            Сила тока в проводнике прямо пропорциональна напряжению и обратно пропорциональна сопротивлению.

            [I=frac{U}{R}]

            Напряжение, Закон Ома, формула

            Падение напряжения на участке проводника равно произведению силы тока в проводнике на сопротивление этого участка.

            [U = IR]

            Напряжение, Закон Ома, формула.

            Падение напряжения на участке проводника равно произведению силы тока в проводнике на сопротивление этого участка.

            Анализ простых схем с помощью закона Ома

            Давайте посмотрим, как эти формулы работают, чтобы помочь нам анализировать простые схемы:


            Рисунок 1 – Пример простой схемы

            В приведенной выше схеме есть только один источник напряжения (батарея слева) и только один источник сопротивления току (лампа справа). Это позволяет очень легко применить закон Ома. Если мы знаем значения любых двух из трех величин (напряжения, тока и сопротивления) в этой цепи, мы можем использовать закон Ома для определения третьей.

            В этом первом примере мы вычислим величину тока (I) в цепи, учитывая значения напряжения (E) и сопротивления (R):


            Рисунок 2 – Пример 1. Известны напряжение источника и сопротивление лампы

            Какая величина тока (I) в этой цепи?

            [I = frac{E}{R} = frac{12 В}{3 Ом} = 4 А]

            Во втором примере мы вычислим величину сопротивления (R) в цепи, учитывая значения напряжения (E) и тока (I):


            Рисунок 3 – Пример 2. Известны напряжение источника и ток в цепи

            Какое сопротивление (R) оказывает лампа?

            [R = frac{E}{I} = frac{36 В}{4 А} = 9 Ом]

            В последнем примере мы рассчитаем величину напряжения, подаваемого батареей, с учетом значений тока (I) и сопротивления (R):


            Рисунок 4 – Пример 3. Известны ток в цепи и сопротивление лампы

            Какое напряжение обеспечивает батарея?

            [E = IR = (2 А)(7 Ом) = 14 В]

            Метода треугольника закона Ома

            Закон Ома – очень простой и полезный инструмент для анализа электрических цепей. Он так часто используется при изучении электричества и электроники, что студент должен запомнить его. Если вы не очень хорошо умеете работать с формулами, то для его запоминания существует простой прием, помогающий использовать его для любой величины, зная две других. Сначала расположите буквы E, I и R в виде треугольника следующим образом:


            Рисунок 5 – Треугольник закона Ома

            Если вы знаете E и I и хотите определить R, просто удалите R с картинки и посмотрите, что осталось:


            Рисунок 6 – Закон Ома для определения R

            Если вы знаете E и R и хотите определить I, удалите I и посмотрите, что осталось:


            Рисунок 7 – Закон Ома для определения I

            Наконец, если вы знаете I и R и хотите определить E, удалите E и посмотрите, что осталось:


            Рисунок 8 – Закон Ома для определения E

            В конце концов, вам придется научиться работать с формулами, чтобы серьезно изучать электричество и электронику, но этот совет может облегчить запоминание ваших первых вычислений. Если вам удобно работать с формулами, всё, что вам нужно сделать, это зафиксировать в памяти E = IR и вывести из нее две другие формулы, когда они вам понадобятся!

            Что дает параллельное и последовательное соединение

            Теоретические знания — это хорошо, но как их применить на практике? Параллельно и последовательно могут соединяться элементы любого типа. Но мы рассматривали только простейшие формулы, описывающие линейные элементы. Линейные элементы — это сопротивления, которые еще называют «резисторы». Итак, вот как можно использовать полученные знания:

            В общем, это наиболее распространенные варианты использования этих соединений.

            Параллельное и последовательное соединение

            В электрике элементы соединяются либо последовательно — один за другим, либо параллельно — это когда к одной точке подключены несколько входов, к другой — выходы от тех же элементов.

            Закон Ома для параллельного и последовательного соединения

            Параллельное соединение

            Параллельное соединение — это когда начала проводников/элементов сходятся в одной точке, а в другой — соединены их концы. Постараемся объяснить законы, которые справедливы для соединений этого типа. Начнем с тока. Ток какой-то величины подается в точку соединения элементов. Он разделяется, протекая по всем проводникам. Отсюда делаем вывод, что общий ток на участке равен сумме тока на каждом из элементов: I = I1 + I2 + I3.

            Теперь относительно напряжения. Если напряжение — это работа по перемещению заряда, тоо работа, которая необходима на перемещение одного заряда будет одинакова на любом элементе. То есть, напряжение на каждом параллельно подключенном элементе будет одинаковым. U = U1=U2=U3. Не так весело и наглядно, как в случае с объяснением закона Ома для участка цепи, но понять можно.

            Для сопротивления все несколько сложнее. Давайте введем понятие проводимости. Это характеристика, которая показывает насколько легко или сложно заряду проходить по этому проводнику. Понятно, что чем меньше сопротивление, тем проще току будет проходить. Поэтому проводимость — G — вычисляется как величина обратная сопротивлению. В формуле это выглядит так: G = 1/R.

            Законы для параллельного соединения

            Для чего мы говорили о проводимости? Потому что общая проводимость участка с параллельным соединением элементов равна сумме проводимости для каждого из участков. G = G1 + G2 + G3 — понять несложно. Насколько легко току будет преодолеть этот узел из параллельных элементов, зависит от проводимости каждого из элементов. Вот и получается, что их надо складывать.

            Теперь можем перейти к сопротивлению. Так как проводимость — обратная к сопротивлению величина, можем получить следующую формулу: 1/R = 1/R1 + 1/R2 + 1/R3.

            Последовательное соединение

            Как работает закон Ома для этих случаев? При последовательном соединении сила тока, протекающая через цепочку элементов, будет одинаковой. Напряжение участка цепи с последовательно подключенными элементами считается как сумма напряжений на каждом участке. Как можно это объяснить? Протекание тока через элемент — это перенос части заряда с одной его части в другую. То есть, это определенная работа. Величина этой работы и есть напряжение. Это физический смысл напряжения. Если с этим понятно, двигаемся дальше.

            При последовательном соединении приходится переносить заряд по очереди через каждый элемент. И на каждом элементе это определенный «объем» работы. А чтобы найти объем работы на всем участке цепи, надо работу на каждом элементе сложить. Вот и получается, что общее напряжение — это сумма напряжений на каждом из элементов.

            Последовательное соединение и параметры этого участка цепи.

            Точно так же — при помощи сложения — находится и общее сопротивление участка цепи. Как можно это себе представить? Ток, протекая по цепочке элементов, последовательно преодолевает все сопротивления. Одно за другим. То есть чтобы найти сопротивление, которое он преодолел, надо сопротивления сложить. Примерно так. Математический вывод более сложен, а так понять механизм действия этого закона проще.

            Практическое использование

            Собственно, к любому участку цепи можно применить этот закон. Пример приведен на рисунке.


            Применяем закон к любому участку цепи.

            Используя такой план, можно вычислить все необходимые характеристики для неразветвленного участка. Рассмотрим более детальные примеры.

            Находим силу тока

            Рассмотрим теперь более определенный пример, допустим, возникла необходимость узнать ток, протекающий через лампу накаливания. Условия:

            • Напряжение – 220 В;
            • R нити накала – 500 Ом.

            Решение задачи будет выглядеть следующим образом: 220В/500Ом=0,44 А.

            Рассмотрим еще одну задачу со следующими условиями:

            • R=0,2 МОм;
            • U=400 В.

            В этом случае, в первую очередь, потребуется выполнить преобразование: 0,2 МОм = 200000 Ом,после чего можно приступать к решению: 400 В/200000 Ом=0,002 А (2 мА).
            Вычисление напряжения
            Для решения мы также воспользуемся законом, составленным Омом. Итак задача:

            • R=20 кОм;
            • I=10 мА.

            Преобразуем исходные данные:

            • 20 кОм = 20000 Ом;
            • 10 мА=0,01 А.

            Решение: 20000 Ом х 0,01 А = 200 В.

            Незабываем преобразовывать значения, поскольку довольно часто ток может быть указан в миллиамперах.

            Сопротивление

            Несмотря на то, что общий вид способа для расчета параметра «R» напоминает нахождение значения «I», между этими вариантами существуют принципиальные различия. Если ток может меняться в зависимости от двух других параметров, то R (на практике) имеет постоянное значение. То есть по своей сути оно представляется в виде неизменной константы.

            Если через два разных участка проходит одинаковый ток (I), в то время как приложенное напряжение (U) различается, то, опираясь на рассматриваемый нами закон, можно с уверенностью сказать, что там где низкое напряжение «R» будет наименьшим. Рассмотрим случай когда разные токи и одинаковое напряжение на несвязанных между собой участках. Согласно закону, составленному Омом, большая сила тока будет характерна небольшому параметру «R».

            Рассмотрим несколько примеров

            Допустим, имеется цепь, к которой подведено напряжение U=50 В, а потребляемый ток I=100 мА. Чтобы найти недостающий параметр, следует 50 В / 0,1 А (100 мА), в итоге решением будет – 500 Ом.

            Вольтамперная характеристика позволяет наглядно продемонстрировать пропорциональную (линейную) зависимость закона. На рисунке ниже составлен график для участка с сопротивлением равным одному Ому (почти как математическое представление закона Ома).

            Изображение вольт-амперной характеристики, где R=1 Ом


            Изображение вольт-амперной характеристики

            Вертикальная ось графика отображает ток I (A), горизонтальная – напряжение U(В). Сам график представлен в виде прямой линии, которая наглядно отображает зависимость от сопротивления, которое остается неизменным. Например, при 12 В и 12 А «R» будет равно одному Ому (12 В/12 А).

            Обратите внимание, что на приведенной вольтамперной характеристике отображены только положительные значения. Это указывает, что цепь рассчитана на протекание тока в одном направлении. Там где допускается обратное направление, график будет продолжен на отрицательные значения.

            Заметим, что оборудование, вольт-амперная характеристика которого отображена в виде прямой линии, именуется — линейным. Этот же термин используется для обозначения и других параметров.

            Помимо линейного оборудования, есть различные приборы, параметр «R» которых может меняться в зависимости от силы тока или приложенного напряжения. В этом случая для расчета зависимости нельзя использовать закон Ома. Оборудование такого типа называется нелинейным, соответственно, его вольт-амперные характеристики не будут отображены в виде прямых линий.

            Видео: Закон Ома для участка цепи — практика расчета цепей

            Источники

            • https://poschitat.online/zakon-oma
            • https://tel-spb.ru/ohm/
            • https://radioprog.ru/post/920
            • https://elektroznatok.ru/info/teoriya/zakon-oma
            • https://www.asutpp.ru/zakon-oma-dlya-uchastka-cepi.html

            Закон ома краткое определение — Вместе мастерим

            Закон Ома, основанный на опытах, представляет собой в электротехнике основной закон, который устанавливает связь силы электрического тока с сопротивлением и напряжением.

            Появление смартфонов, гаджетов, бытовых приборов и прочей электротехники коренным образом изменило облик современного человека. Приложены огромные усилия, направленные на исследование физических закономерностей для улучшения старой и создания новой техники. Одной из таких зависимостей является закон Ома.

            Закон Ома – полученный экспериментальным путём (эмпирический) закон, который устанавливает связь силы тока в проводнике с напряжением на концах проводника и его сопротивлением, был открыт в 1826 году немецким физиком-экспериментатором Георгом Омом.

            Строгая формулировка закона Ома может быть записана так: сила тока в проводнике прямо пропорциональна напряжению на его концах (разности потенциалов) и обратно пропорциональна сопротивлению этого проводника.

            Формула закона Ома записывается в следующем виде:

            U – электрическое напряжение (разность потенциалов), единица измерения напряжения- вольт [В];

            Согласно закону Ома, увеличение напряжения, например, в два раза при фиксированном сопротивлении проводника, приведёт к увеличению силы тока также в два раза

            И напротив, уменьшение тока в два раза при фиксированном напряжении будет означать, что сопротивление увеличилось в два раза.

            Рассмотрим простейший случай применения закона Ома. Пусть дан некоторый проводник сопротивлением 3 Ом под напряжением 12 В. Тогда, по определению закона Ома, по данному проводнику течет ток равный:

            Существует мнемоническое правило для запоминания этого закона, которое можно назвать треугольник Ома. Изобразим все три характеристики (напряжение, сила тока и сопротивление) в виде треугольника. В вершине которого находится напряжение, в нижней левой части – сила тока, а в правой – сопротивление.

            Правило работы такое: закрываем пальцем величину в треугольнике, которую нужно найти, тогда две оставшиеся дадут верную формулу для поиска закрытой.

            Где и когда можно применять закон Ома?

            Закон Ома в упомянутой форме справедлив в достаточно широких пределах для металлов. Он выполняется до тех пор, пока металл не начнет плавиться. Менее широкий диапазон применения у растворов (расплавов) электролитов и в сильно ионизированных газах (плазме).

            Работая с электрическими схемами, иногда требуется определять падение напряжения на определенном элементе. Если это будет резистор с известной величиной сопротивления (она проставляется на корпусе), а также известен проходящий через него ток, узнать напряжение можно с помощью формулы Ома, не подключая вольтметр.

            Значение Закона Ома

            Закон Ома определяет силу тока в электрической цепи при заданном напряжении и известном сопротивлении.

            Он позволяет рассчитать тепловые, химические и магнитные действия тока, так как они зависят от силы тока.

            Закон Ома является чрезвычайно полезным в технике(электронной/электрической), поскольку он касается трех основных электрических величин: тока, напряжения и сопротивления. Он показывает, как эти три величины являются взаимозависимыми на макроскопическом уровне.

            Если бы было можно охарактеризовать закон Ома простыми словами, то наглядно это выглядело бы так:

            Из закона Ома вытекает, что замыкать обычную осветительную сеть проводником малого сопротивления опасно. Сила тока окажется настолько большой, что это может иметь тяжелые последствия.

            Немецкий физик Георг Симон Ом (1787—1854) открыл основной закон электрической цепи.

            Закон Ома для участка цепи:

            Определение: Cила тока I на участке электрической цепи прямо пропорциональна напряжению U на концах участка и обратно пропорциональна его сопротивлению R.

            1. I — сила тока (в системе СИ измеряется — Ампер)
              • Сила тока в проводнике прямо пропорциональна напряжению и обратно пропорциональна сопротивлению.
              • Формула: I=frac
              • U — напряжение (в системе СИ измеряется — Вольт)

            Падение напряжения на участке проводника равно произведению силы тока в проводнике на сопротивление этого участка.

            Формула: U=IR

          • R— электрическое сопротивление (в системе СИ измеряется — Ом).
            • Электрическое сопротивление R это отношение напряжения на концах проводника к силе тока, текущего по проводнику.
            • Формула R=frac
            • Определение единицы сопротивления — Ом

              1 Ом представляет собой электрическое сопротивление участка проводника, по которому при напряжении 1(Вольт) протекает ток 1 (Ампер).

              Закон Ома для полной цепи

              Определение: Сила тока в цепи пропорциональна действующей в цепи ЭДС и обратно пропорциональна сумме сопротивлений цепи и внутреннего сопротивления источника

              Формула I=frac <varepsilon>

              • varepsilon — ЭДС источника напряжения, В;
              • I — сила тока в цепи, А;
              • R — сопротивление всех внешних элементов цепи, Ом;
              • r — внутреннее сопротивление источника напряжения, Ом.

              Как запомнить формулы закона Ома

              Треугольник Ома поможет запомнить закон. Нужно закрыть искомую величину, и два других символа дадут формулу для её вычисления.

              .

              • U — электрическое напряжение;
              • I — сила тока;
              • P — электрическая мощность;
              • R — электрическое сопротивление

              Смотри также:

              Для закрепления своих знаний решай задания и варианты ЕГЭ по физике с ответами и пояснениями.

              Говорят: «не знаешь закон Ома – сиди дома». Так давайте же узнаем (вспомним), что это за закон, и смело пойдем гулять.

              Основные понятия закона Ома

              Как понять закон Ома? Нужно просто разобраться в том, что есть что в его определении. И начать следует с определения силы тока, напряжения и сопротивления.

              Сила тока I

              Пусть в каком-то проводнике течет ток. То есть, происходит направленное движение заряженных частиц – допустим, это электроны. Каждый электрон обладает элементарным электрическим зарядом (e= -1,60217662 × 10 -19 Кулона). В таком случае через некоторую поверхность за определенный промежуток времени пройдет конкретный электрический заряд, равный сумме всех зарядов протекших электронов.

              Отношение заряда к времени и называется силой тока. Чем больший заряд проходит через проводник за определенное время, тем больше сила тока. Сила тока измеряется в Амперах.

              Напряжение U, или разность потенциалов

              Это как раз та штука, которая заставляет электроны двигаться. Электрический потенциал характеризует способность поля совершать работу по переносу заряда из одной точки в другую. Так, между двумя точками проводника существует разность потенциалов, и электрическое поле совершает работу по переносу заряда.

              Физическая величина, равная работе эффективного электрического поля при переносе электрического заряда, и называется напряжением. Измеряется в Вольтах. Один Вольт – это напряжение, которое при перемещении заряда в 1 Кл совершает работу, равную 1 Джоуль.

              Сопротивление R

              Ток, как известно, течет в проводнике. Пусть это будет какой-нибудь провод. Двигаясь по проводу под действием поля, электроны сталкиваются с атомами провода, проводник греется, атомы в кристаллической решетке начинают колебаться, создавая электронам еще больше проблем для передвижения. Именно это явление и называется сопротивлением. Оно зависит от температуры, материала, сечения проводника и измеряется в Омах.

              Памятник Георгу Симону Ому

              Формулировка и объяснение закона Ома

              Закон немецкого учителя Георга Ома очень прост. Он гласит:

              Сила тока на участке цепи прямо пропорционально напряжению и обратно пропорциональна сопротивлению.

              Георг Ом вывел этот закон экспериментально (эмпирически) в 1826 году. Естественно, чем больше сопротивление участка цепи, тем меньше будет сила тока. Соответственно, чем больше напряжение, тем и ток будет больше.

              Кстати! Для наших читателей сейчас действует скидка 10% на любой вид работы

              Данная формулировка закона Ома – самая простая и подходит для участка цепи. Говоря «участок цепи» мы подразумеваем, что это однородный участок, на котором нет источников тока с ЭДС. Говоря проще, этот участок содержит какое-то сопротивление, но на нем нет батарейки, обеспечивающей сам ток.

              Если рассматривать закон Ома для полной цепи, формулировка его будет немного иной.

              Пусть у нас есть цепь, в ней есть источник тока, создающий напряжение, и какое-то сопротивление.

              Закон запишется в следующем виде:

              Объяснение закона Ома для полой цепи принципиально не отличается от объяснения для участка цепи. Как видим, сопротивление складывается из собственно сопротивления и внутреннего сопротивления источника тока, а вместо напряжения в формуле фигурирует электродвижущая сила источника.

              Кстати, о том, что такое что такое ЭДС, читайте в нашей отдельной статье.

              Как понять закон Ома?

              Чтобы интуитивно понять закон Ома, обратимся к аналогии представления тока в виде жидкости. Именно так думал Георг Ом, когда проводил опыты, благодаря которым был открыт закон, названный его именем.

              Представим, что ток – это не движение частиц-носителей заряда в проводнике, а движение потока воды в трубе. Сначала воду насосом поднимают на водокачку, а оттуда, под действием потенциальной энергии, она стремиться вниз и течет по трубе. Причем, чем выше насос закачает воду, тем быстрее она потечет в трубе.

              Отсюда следует вывод, что скорость потока воды (сила тока в проводе) будет тем больше, чем больше потенциальная энергия воды (разность потенциалов)

              Сила тока прямо пропорциональна напряжению.

              Теперь обратимся к сопротивлению. Гидравлическое сопротивление – это сопротивление трубы, обусловленное ее диаметром и шероховатостью стенок. Логично предположить, что чем больше диаметр, тем меньше сопротивление трубы, и тем большее количество воды (больший ток) протечет через ее сечение.

              Сила тока обратно пропорциональна сопротивлению.

              Такую аналогию можно проводить лишь для принципиального понимания закона Ома, так как его первозданный вид – на самом деле довольно грубое приближение, которое, тем не менее, находит отличное применение на практике.

              В действительности, сопротивление вещества обусловлено колебанием атомов кристаллической решетки, а ток – движением свободных носителей заряда. В металлах свободными носителями являются электроны, сорвавшиеся с атомных орбит.

              Ток в проводнике

              В данной статье мы постарались дать простое объяснение закона Ома. Знание этих на первый взгляд простых вещей может сослужить Вам неплохую службу на экзамене. Конечно, мы привели его простейшую формулировку закона Ома и не будем сейчас лезть в дебри высшей физики, разбираясь с активным и реактивным сопротивлениями и прочими тонкостями.

              Если у Вас возникнет такая необходимость, Вам с удовольствием помогут сотрудники нашего студенческого сервиса. А напоследок предлагаем Вам посмотреть интересное видео про закон Ома. Это действительно познавательно!

              Закон Ома | Физика

              В предыдущих параграфах были рассмотрены три величины, характеризующие протекание электрического тока в цепи,— сила тока I, напряжение U и сопротивление R. Между этими величинами существует определенная связь. Закон, выражающий эту связь, был установлен в 1827 г. немецким ученым Г. Омом и поэтому носит его имя.

              Выделим в произвольной электрической цепи участок, обладающий сопротивлением R и находящийся под напряжением U (рис. 37). Согласно закону Ома:
              Сила тока на участке цепи равна отношению напряжения на этом участке к его сопротивлению.

              Математически закон Ома записывается в виде следующей формулы:

              I = U/R      (14.1)

              Закон Ома позволяет установить, что будет происходить с силой тока на участке цепи при изменении его сопротивления или напряжения.

              1. При неизменном сопротивлении сила тока прямо пропорциональна напряжению: чем больше напряжение U на концах участка цепи, тем больше сила тока I на этом участке. Увеличив (или уменьшив) напряжение в несколько раз, мы во столько же раз увеличим (или уменьшим) силу тока.

              Проиллюстрируем эту закономерность на опыте. Соберем электрическую цепь из источника тока, лампы, амперметра и ключа (рис. 38, а). В качестве источника тока будем использовать устройство, позволяющее регулировать выходное напряжение от 4 до 12 В. Измеряя силу тока в цепи при разных напряжениях, можно убедиться в том, что она действительно пропорциональна напряжению.

              2. При неизменном напряжении сила тока обратно пропорциональна сопротивлению: чем больше сопротивление R участка цепи, тем меньше сила тока I в нем.

              Для проверки этой закономерности заменим в используемой цепи лампу на магазин сопротивлений (рис. 38, б). Измеряя силу тока при разных сопротивлениях, мы увидим, что сила тока I и сопротивление R действительно находятся в обратно пропорциональной зависимости.

              При уменьшении сопротивления сила тока возрастает. Если сила тока превысит допустимое для данной цепи значение, включенные в нее приборы могут выйти из строя; провода при этом могут раскалиться и стать причиной пожара. Именно такая ситуация возникает при коротком замыкании. Так называют соединение двух точек электрической цепи, находящихся под некоторым напряжением, коротким проводником, обладающим очень малым сопротивлением.

              Короткое замыкание может возникнуть при соприкосновении оголенных проводов, при небрежном ремонте проводки под током, при большом скоплении пыли на монтажных платах и даже при случайном попадании какого-нибудь насекомого внутрь прибора.

              На законе Ома основан экспериментальный способ определения сопротивления. Из формулы (14.1) следует, что

              R = U/I      (14.2)

              Поэтому для нахождения сопротивления R участка цепи надо измерить на нем напряжение U, затем силу тока I, после чего разделить первую из этих величин на вторую. Соответствующая этому схема цепи изображена на рисунке 39.

              Если, наоборот, известны сопротивление R и сила тока I на участке цепи, то закон Ома позволяет рассчитать напряжение U на его концах. Из формулы (14.1) получаем

              U = IR     (14.3)

              Чтобы найти напряжение U на концах участка цепи, надо силу тока I на этом участке умножить на его сопротивление R.

              Опубликовав книгу, в которой излагался открытый им закон «Теоретические исследования электрических цепей», Георг Ом написал, что «рекомендует ее добрым людям с теплым чувством отца, не ослепленного обезьяньей любовью к детям, но довольствующегося указанием на открытый взгляд, с которым его дитя смотрит на злой мир». Мир действительно оказался для него злым, и уже через год после выхода его книги в одном из журналов появилась статья, в которой работы Ома были подвергнуты уничтожающей критике. «Тот, кто благоговейными глазами взирает на вселенную,— говорилось в статье,— должен отвернуться от этой книги, являющейся плодом неисправимых заблуждений, преследующих единственную цель — умалить величие природы».

              Злобные и безосновательные нападки на Ома не прошли бесследно. Теорию Ома не приняли. И вместо продолжения научных исследований он должен был тратить время и энергию на полемику со своими оппонентами. В одном из своих писем Ом написал: «Рождение «Электрических цепей» принесло мне невыразимые страдания, и я готов проклясть час их зарождения».

              Но это были временные трудности. Постепенно, сначала в России, а затем и в других странах, теория Ома получила полное признание. Закон Ома внес такую ясность в правила расчета токов и напряжений в электрических цепях, что американский ученый Дж. Генри, узнав об открытиях Ома, не удержался от восклицания: «Когда я первый раз прочел теорию Ома, то она мне показалась молнией, вдруг осветившей комнату, погруженную во мрак».

              ??? 1. Сформулируйте закон Ома. 2. Как изменится сила тока на участке цепи, если при неизменном сопротивлении увеличить напряжение на его концах? 3. Как изменится сила тока, если при неизменном напряжении увеличить сопротивление участка цепи? 4. Как с помощью вольтметра и амперметра можно измерить сопротивление проводника? 5. По какой формуле находится напряжение, если известны сила тока и сопротивление данного участка? 6. Что называют коротким замыканием? Почему при этом увеличивается сила тока? 7. Объясните причину короткого замыкания в ситуациях, изображенных на рисунке 40.

              «Закон Ома для участка цепи» (8-й класс)

              Цель урока: раскрыть взаимозависимость силы тока, напряжения и сопротивления на участке электрической цепи.

              Образовательная:

              • знакомство с новым материалом, работа по формированию навыков (сравнение физических величин, построение графиков),
              • усвоить зависимость между физическими величинами,
              • закрепление знаний и умений.

              Развивающая:

              • развивать умения наблюдать, сопоставлять, сравнивать и обобщать результаты экспериментов,
              • продолжить формирование умений пользоваться теоретическими и экспериментальными методами физической науки для обоснования выводов по изучаемой теме и для решения задач.

              Воспитательная: Развивать познавательный интерес к предмету.

              Этап урока Деятельность учителя Деятельность учащихся Методическое обоснование
              1 Организационный (1-2 мин.)
                1. Учитель приветствует учащихся.

              2. Учитель выявляет отсутствующих, выясняет причину отсутствия.

              3. Проверка готовности учащихся к уроку (внешний вид, рабочая поза, состояние рабочего места).

              4. Проверка подготовленности классного помещения к уроку (чистая доска, мел, тряпка, порядок в классе).

              5. Организация внимания.

              1. Учащиеся встают для приветствия учителя.

              2. Дежурные называют отсутствующих

              3. Учащиеся настраиваются на рабочий ритм урока.

              1. Задача этого этапа: обеспечить благоприятную внешнюю обстановку для работы на уроке и психологически подготовить учащихся к общению и предстоящему занятию.

              2. Служит для воспитания внимательного отношения к окружающим, друг к другу, учебной дисциплины.

              3. Этот момент формирует эталон внешности, привычку держать свое рабочее место в порядке.

              4. Формирование потребности в том, чтобы рабочее место класса и учителя было полностью подготовлено к работе.

              5. Настройка на рабочий ритм в организационный момент способствует созданию необходимого делового и психологического контакта между учителем и учащимися.

              2. Этап подготовки учащихся к активному, сознательному усвоению материала.

              (3-4 мин.)

                До начала урока на доске записывается тема “Закон Ома для участка цепи”.

              Доска распланирована учителем, и все основные моменты необходимые для урока отражены на ней.

              Цель урока: раскрыть взаимозависимость силы тока, напряжения и сопротивления на участке электрической цепи.

              Учащиеся записывают в тетрадь число, тему урока.

              Учащиеся слушают учителя.

              Используется словесно- иллюстративный метод.

              Мотивация к учебно-познавательной деятельности.

              3 Повторение.

              (7мин)

               

               

               

               

               

               

               

               

               

               

               

               

               

               

               

               

               

               

               

               

               

               

               

               

               

               

               

               

               

               

               

               

               

               

               

               

               

               

               

              3.

              Повторение мы начнем с решения №1267 (страница146).

              По графику зависимости силы тока в проводнике от напряжения (рисунок. 310) определите, чему равна сила тока в проводнике при напряжении 2; 1; 5; 6; 10В.

              Учитель вызывает ученика к доске.

              График и таблица заранее нарисованы учителем на доске.

              Таблица1.

              1. Во сколько раз уменьшается напряжение?

              2. Что в данном случае произойдет с силой тока?

              3. Что представляет собой прямо пропорциональная зависимость?

              4.Что является графиком этой

              зависимости?

              5. Каким уравнением в алгебре представляется эта зависимость?

              6.В нашем случае, какая физическая величина является осью OY?

              7. Какая физическая величина является осью ОХ?

              8. Какому числовому значению равен коэффициент k в данном уравнении?

              9.Каким уравнением выражается зависимость изучаемого графика?

              10. Если напряжение равно 5В, то какому числовому значению равна сила тока?

              11. Во сколько раз увеличивается напряжение?

              12. Что происходит с силой тока?

              Парная работа учащихся. Первый ряд выполняет задание для 4В,8В. Второй ряд для 6В,10В. Третий ряд выполняет задание для 4В,8В. Определяют силу тока в проводнике.

              Через 1 минуту ученики проверяют решения.

              Учитель еще раз с помощью таблицы

              разбирает прямо пропорциональную зависимость силы тока от напряжения при постоянном сопротивлении.

              2.На доске и на раздаточном материале изображена зависимость силы тока от напряжения для двух проводников.

              1.Какой из проводников имеет большую силу тока?

              2. Как это утверждение можно доказать?

              Объяснение нового материала (20 минут)

              Ребята, мы с вами рассмотрели, как сила тока зависит от напряжения. Давайте вместе попробуем ответить на вопрос.

              1.Как сила тока зависит от сопротивления проводника, при постоянном напряжении на его концах?

              U = 2В

              Проверим это на опыте. Учитель демонстрирует зависимость силы тока от сопротивления при постоянном напряжении на концах проводника.

              Обратите внимание на демонстрационную установку. Учитель вместе с учениками раскрывает назначение каждого элемента собранной демонстрационной установки, определяя цену деления электроизмерительных приборов.

              Включают на магазине сопротивлений 4 Ом, замыкают цепь. Регулируя сопротивление реостата, добиваются на зажимах магазина напряжение 2В; при этом амперметр покажет 0.5А. Подготовленную таким способом установку, демонстрируют учащимся: включают ток и измеряют. Затем (уменьшают) сопротивление магазина и, доводя каждый раз с помощью реостата, напряжение до прежней величины (2В), измеряют силу тока.

              4. Какой вывод можно сделать по данным этой таблицы?

              Давайте построим график обратно пропорциональной зависимости силы тока и сопротивления.

              5. Как называется этот график?

              6.Как записывается уравнение гиперболы?

              7. Какая физическая величина на данном графике выполняет ось ОУ?8. Какая физическая величина выполняет ось Ох?

              9. Как записывается уравнение гиперболы для графика зависимости силы тока от сопротивления при постоянном напряжении?

              10. Какому числовому значению равен коэффициент К?

              Давайте, определим коэффициент, подставив значения силы тока и сопротивления.

              Давайте проверим правильность составленного уравнения.

              11. Какой физической величине равен коэффициент К?

              Если мы в уравнение подставим вместо коэффициента физическую величину напряжение, то, что получим?

              Учитель объясняет учащимся, что благодаря эксперименту и математическим выкладкам получился закон Ома для участка цепи.

              Учитель просит сформулировать закон Ома для участка цепи учащихся.

              Учитель еще раз формулирует закон Ома для участка цепи.

              12. Как из закона Ома выразить напряжение? Если учащиеся затрудняются в ответе. Учитель возвращает учащихся к таблице и помогает наводящими вопросами. Давайте подставим вместо цифр физические величины напряжение, силу тока, сопротивление. (Устно)

              Давайте, выразим из закона Ома для участка цепи сопротивление. В случае затруднения учитель помогает наводящими вопросами.

              15. Какую цель мы ставили в начале урока?

              16. Справились ли мы с поставленной целью?

              Учитель ждет ответ учащихся?

              При оформлении задачи учащиеся ответы отмечают на графике и записывают в таблицу, где третья колонка пока рассматривается, как значение коэффициента К. После изучения новой темы учащиеся докажут, что К это сопротивление.

              U=2В I=0.5А U=1В I=0.25А

              График 1.

              1. Напряжение уменьшается в 2 раза.

              2. Сила тока уменьшается в 2 раза, так как она находится в прямо пропорциональной зависимости от напряжения.

              3. Если две физические величины находятся в прямо пропорциональной зависимости, то при увеличении одой физической величины в k раз, другая величина так же увеличивается в k раз и наоборот.

              4. Прямая линия.

              5.у=kх

              6. Роль оси ОУ выполняет на графике напряжение.

              7. Роль оси ОХ выполняет на графике сила тока.

              8.U=k*I

              k=U /I

              k=2/0.5=4

              9. I=U\4

              Учащиеся с помощью карточек составляют это уравнение.

              10. U=5В

              I=5/4=1.25А

              11. В5 раз.

              12. Сила тока тоже увеличивается в 5 раз.

              Учащиеся, не справившиеся с этим заданием, слушают объяснение. В том случае, если ученик у доски не справился с заданием, ему помогает класс.

              U,В I,А К
              2 0.5  
              1 0.25  
              5 1.25  
              4 1  
              8 2  
              6 1.5  
              10 2.5  

              График 2.

              Графики перечерчивают в тетрадь.

              1. В первом проводнике сила тока больше.

              2. Если на оси напряжения взять произвольную точку и провести перпендикуляр к 1и 2 графику, а из точки пересечения перпендикуляра и графиков опустить перпендикуляр на ось силы тока. По рисунку видно, что сила тока в первом проводнике больше силы тока во втором проводнике.

              График 3.

              На этом этапе проводится работа с карточками. Каждый учащийся включается в работу, поднимая карточку с ответом.

              Учащиеся собирают демонстрационную установку на парте с помощью раздаточного материала. Приложение 1.

              Источник тока – для создания и поддержания электрического поля в проводнике. Амперметр для измерения силы тока в проводнике. Вольтметр для измерения напряжения на концах проводника. Демонстрационный магазин сопротивления – для изменения сопротивления на участке цепи. Ключ – (замыкающее и размыкающее устройство), нужен для включения и выключения в нужное время источника тока.

              Соединительные провода – доставляют электрическую энергию в электрическую цепь.

              Ученики следят за показаниями вольтметра. Напряжение на концах проводника во время опыта постоянно.

              Одновременно, определяют силу тока в цепи и записывают результаты в таблицу 2.

              4.Между силой тока и сопротивлением существует обратно пропорциональная связь. Если сила тока увеличивается, то сопротивление уменьшается и наоборот.

              Один ученик строит график обратно пропорциональной зависимости силы тока и сопротивления у доски.

              Остальные учащиеся работают в тетради. График 4.

              5. Гипербола.

              6. У=К/Х

              7. Роль оси ОУ выполняет на графике сила тока.

              8. Роль оси ОХ выполняет на графике сопротивление.

              9. I=К/R

              К=I*R

              К=2*1=2

              К=2

              U=2В

              Учащиеся в тетради записывают зависимость от трех физических величин силы тока, напряжения, сопротивления. Составляют эту зависимость с помощью карточек

              I=U/R

              Учащиеся вводят теоретическую формулировку закона Ома для участка цепи.

              “Сила тока в участке цепи прямо пропорциональна напряжению на концах этого участка и обратно пропорциональна его сопротивлению”.

              Учащиеся еще раз проговаривают его про себя.

              U=I*R

              Напряжение прямо пропорционально силе тока при постоянном сопротивлении.

              U=I*R

              R=U/I

              Сопротивление обратно пропорционально силе тока при постоянном напряжении.

              Раскрыть взаимозависимость силы тока, напряжения и сопротивления на участке электрической цепи.

              Используется метод синтезирующей беседы нацеленной на систематизацию знаний и способов их применения в нестандартных ситуациях, на перенос их в решении проблем на межпредметной основе.

              Форма познавательной деятельности учащихся фронтальная.

              Использование модели прямоугольной системы координат позволяет учителю сэкономить время при повторении материала, кроме того, у учащихся работает одновременно зрительная и слуховая память, а картинка привлекает внимание, что делает восприятие и закрепление более эффективным.

              Учебно-воспитательная задача этого этапа:

              при решении данной задачи повторить и отработать навыки по вычислению силы тока и напряжения,

              находящихся в прямо пропорциональной зависимости, при постоянном сопротивлении.

              Учащиеся учатся работать с графиком, определять по известному значению силы тока напряжение, а так же отмечать координаты.

              Интеграция физики с математикой позволяет на основе алгебраического уравнения прямо пропорциональной зависимости вывести физическое уравнение зависимости силы тока от напряжения при постоянном сопротивлении. Практически они записывают закон Ома для участка цепи, не зная что коэффициент К это и есть сопротивление, но этот момент они раскроют после изучения новой темы.

               

              Используется наглядный метод иллюстраций во взаимосвязи со словесным

              и практическим

              методом обучения, а предназначается для наглядно-чувственного ознакомления учащихся в виде символьного изображения.

              Используется репродуктивный метод. Разработка вопросов позволяет использовать полученные умения и навыки.

              Актуализируются ранее усвоенные знания, концентрируется внимание, раскрываются потенциальные и реальные возможности учащихся. На основе вводной беседы выясняется степень понимания и готовность учащихся к познанию нового материала.

              Используется практический метод, способствующий развитию логического мышления, памяти, речи учащихся.

              Отражается целенаправленная учебная деятельность, когда каждый ученик и класс в целом объединяются одной целью.

               

              Используется проблемный метод, в котором учитель ставит перед учащимися проблему и сам показывает путь ее решения, вскрывая возникающие противоречия. Суть применения этого метода состоит в том, чтобы показать образец решения проблем. Учащиеся получают эталон научного мышления и познания, образец культуры развертывания познавательных действий.

              Происходит сочетание проблемного метода и наглядного, что позволяет учителю увидеть работу класса и ученика в отдельности. Ученик может ответить на вопрос, а в случае ошибки исправить себя.

              Опыт проводится с набором приборов, которые позволяют в полной мере раскрыть “ Закон Ома для участка цепи”. Определяет однозначность, определенность, истинность. Создаются условия хорошей видимости. Предусматривает эстетичность, это изящное, красивое оформление установки и рациональное выполнение опыта.

              Эмоциональность отражает результат воздействия демонстрируемого опыта на психику учащихся, она выражается в том впечатление, которое оказывает демонстрация.

              Использования графического воспроизводящего

              упражнения помогает учащимся лучше воспринимать, осмысливать и запоминать учебный материал, способствует развитию пространственного воображения.

              Применяется метод, беседа-сообщение который предполагает включение ученика в сам процесс активного участия в добывание новых знаний, в поиск способов их получения, формирования собственных ответов на поставленные учителем вопросы. В ходе эвристической беседы учитель, опираясь на имеющиеся знания и практический опыт, подводит их к пониманию и усвоению новых знаний, формированию правил и выводов. В результате такой совместной деятельности учащиеся приобретают новые знания, путем собственных усилий, размышлений.

              Фронтальный опрос. Отрабатываются основные навыки по “Закону Ома для участка цепи”.

              Происходит восприятие, осмысление и запоминание учащимися готовых научных выводов.

              Воспроизведение, систематизация и отработка базовых знаний. Развитие речи учащихся.

              4 Закрепление изученного материала. (!3минут)
                Учитель возвращает ребят к №1267. Давайте посмотрим на график и найдем сопротивление для разных участков.

              Учитель просит назвать получившееся сопротивление и проверить решение ученика у доски.

              1. Какому значению равно сопротивление.

              2. Какую роль играет это число в уравнение?

              Учитель обращает внимание на уравнение. Предлагает учащимся заменить число 4 сопротивлением.

              3.Что мы получили?

              4. Вернемся ко второму графику. На рисунке дана зависимость силы тока от напряжения для двух проводников. Какой из проводников имеет большее сопротивление?

              5. Самостоятельная работа №1277(страница 147)

              По графикам зависимости силы тока от напряжения определите сопротивление каждого проводника.

              1вариант выполняет задание для 1и 3 графика.

              2 вариант выполняет задание для 2 и 3 графика.

              Учитель через 5 минут показывает решение вариантов, которое заранее было написано за “ушами” доски.

              Учитель еще раз перепроверяет решение и выставляет окончательную оценку.

              Объяснит ее на следующем уроке.

              Учитель вызывает ученика к доске.

              Он находит сопротивление для первого участка. Первый ряд для второго участка, второй ряд для третьего участка, третий ряд для четвертого участка.

              Учащиеся с каждого ряда проговаривают свое решение и правильный ответ.

              R=U/I

              4

              Это коэффициент.

              I=U\4

              I=U\R

              Закон Ома для участка цепи. Сила тока прямо пропорциональна напряжению при постоянном сопротивлении.

              Мы выяснили, что сила тока у первого проводника больше, чем сила тока у второго проводника. Значит, сопротивление первого проводника будет меньше.


              U,В

              График 5.

              Через 5 минут меняются тетрадями

              и проверяют решения друг друга. Выставляют оценку на свое усмотрение.

              1 вариант:

              R1=4/2=2 Ом

              R3=2/2=1 Ом

              2 вариант:

              R2=4/1=4Ом

              R3=2/2=1Ом

              Этот метод позволяет самостоятельно решить задание, проверить его и найти ошибку или убедится в правильности решения ученика у доски.

              Задание способствует развитию внимания, наблюдательности, активизирует деятельность учащихся.

              Используется метод письменного текущего контроля. Позволяет оценить результат обучения на данном уроке. Проверка задачи дает возможность учащимся увидеть свои ошибки и не допустить их при выполнении домашнего задания.

              6

               

              Этап подведения итогов урока. (2 минуты)
                Учитель просит учащихся вспомнить, какую цель поставил перед ними учитель в начале урока.

              Справились ли мы с поставленной целью?

              Учитель ждет ответ учащихся.

              Учитель вместе с учащимися делает вывод, что теперь они готовы к решению задач по данной теме.

              Учитель подводит итог о работе класс, и о работе отдельных учащихся.

              Дает домашнее задание:

              №44 Упражнение19 (1,4,7)

              Спасибо за урок!

              Раскрыть взаимозависимость силы тока, напряжения и сопротивления на участке электрической цепи.

              Учащиеся делают выводы о проделанной работе на уроке, о тех практических навыках, которые они получили в процессе работы.

              Учащиеся записывают домашнее задание в дневник.

              Ученики встают.

              Подведение итогов урока вместе с учащимися формирует привычку подводить итоги своей работы, анализировать свою деятельность.

              Домашнее задание содержит три уровня сложности:

              Упраж19(1)-1 уровень

              Упраж19(4)-2 уровень

              Упраж19(7)-3уровень

              Это позволяет слабым учащимся решить хотя бы 1 уровень сложности и попробовать свои силы в решение более сложных задач.

              Закон Ома и взаимосвязь V-I-R

              В физике есть определенные формулы, настолько мощные и всеобъемлющие, что они достигли уровня общеизвестности. Студент-физик столько раз записывал такие формулы, что запоминал их, даже не пытаясь. Конечно, для профессионалов в этой области такие формулы настолько важны, что они запечатлеваются в их сознании. В области современной физики есть E = m • c 2 . В области ньютоновской механики есть F нетто = м • а. В области волновой механики есть v = f • λ. А в области тока электричества есть ΔV = I • R.

              Преобладающее уравнение, которое пронизывает изучение электрических цепей, это уравнение двух точках цепи ( ΔV ) эквивалентно произведению тока между этими двумя точками ( I ) и общего сопротивления всех электрических устройств между этими двумя точками ( Р ). В оставшейся части этого раздела «Класс физики» это уравнение станет наиболее распространенным уравнением, которое мы видим. Это уравнение, часто называемое уравнением закона Ома , является мощным предсказателем взаимосвязи между разностью потенциалов, током и сопротивлением.

               

               

              Закон Ома как показатель силы тока и сопротивление известно. Тем не менее, хотя это уравнение служит мощным рецептом решения проблем, оно представляет собой гораздо больше. Это уравнение указывает две переменные, которые могут повлиять на величину тока в цепи. Сила тока в цепи прямо пропорциональна разности электрических потенциалов на ее концах и обратно пропорциональна общему сопротивлению внешней цепи. Чем больше напряжение батареи (т. е. разность электрических потенциалов), тем больше ток. И чем больше сопротивление, тем меньше ток. Заряд течет с наибольшей скоростью, когда напряжение батареи увеличивается, а сопротивление уменьшается. В самом деле, двукратное увеличение напряжения батареи приведет к двукратному увеличению тока (если все остальные факторы остаются равными). А увеличение сопротивления нагрузки в два раза приведет к уменьшению тока в два раза до половины его первоначального значения.

              В таблице ниже эта взаимосвязь иллюстрируется качественно и количественно для нескольких цепей с различными напряжениями и сопротивлениями аккумуляторов.

               
              Цепь
              Схема
              Аккумулятор
              Напряжение
              (ΔV)
              Всего
              Сопротивление
              ()
              Текущий
              (Ампер)
              1.
              1,5 В
              3
              0,50 А
              2.
              3,0 В
              3 Ом
              1 А
              3.
              4,5 В
              3
              1,5 А
              4.
              1,5 В
              6
              0,25 А
              5.
              3,0 В
              6
              0,5 А
              6.
              4,5 В
              6 Ом
              0,75 А
              7.
              4,5 В
              9 Ом
              0,50 А


              Строки 1, 2 и 3 показывают, что удвоение и утроение напряжения батареи приводит к удвоению и утроению тока в цепи. Сравнение строк 1 и 4 или строк 2 и 5 показывает, что удвоение общего сопротивления позволяет вдвое уменьшить ток в цепи.

              Поскольку на ток в цепи влияет сопротивление, резисторы часто используются в цепях электроприборов, чтобы влиять на величину тока, присутствующего в его различных компонентах. Увеличивая или уменьшая величину сопротивления в конкретной ветви цепи, производитель может увеличивать или уменьшать величину тока в этой ветви . Кухонные приборы, такие как электрические смесители и регуляторы освещенности, работают, изменяя ток на нагрузке путем увеличения или уменьшения сопротивления цепи. Нажатие различных кнопок на электрическом миксере может изменить режим с смешивания на взбивание, уменьшив сопротивление и позволив большему току в миксере. Точно так же поворот диска на диммерном переключателе может увеличить сопротивление его встроенного резистора и, таким образом, уменьшить ток.

              На приведенной ниже схеме изображена пара цепей, содержащих источник напряжения (батарейный блок), резистор (лампочка) и амперметр (для измерения силы тока). В какой цепи лампочка имеет наибольшее сопротивление? Нажмите кнопку «Просмотреть ответ», чтобы убедиться, что вы правы.


               

               

              Уравнение закона Ома часто изучается в физических лабораториях с использованием резистора, аккумуляторной батареи, амперметра и вольтметра. Амперметр — это прибор, используемый для измерения силы тока в заданном месте. Вольтметр — это устройство, оснащенное щупами, которые можно прикоснуться к двум точкам цепи, чтобы определить разность электрических потенциалов в этих точках. Изменяя количество элементов в аккумуляторной батарее, можно изменять разность электрических потенциалов во внешней цепи. Вольтметр можно использовать для определения этой разности потенциалов, а амперметр можно использовать для определения тока, связанного с этим ΔV. Батарея может быть добавлена ​​к блоку батарей, и процесс может быть повторен несколько раз, чтобы получить набор данных I-ΔV. График зависимости I от ΔV даст линию с наклоном, эквивалентным обратной величине сопротивления резистора. Это можно сравнить с заявленным производителем значением, чтобы определить точность лабораторных данных и достоверность уравнения закона Ома.

               

              Величины, символы, уравнения и единицы!

              Склонность обращать внимание на единицы измерения — неотъемлемая черта любого хорошего студента-физика. Многие трудности, связанные с решением задач, могут быть связаны с неспособностью уделить внимание единицам. По мере того, как все больше и больше электрических величин и соответствующих им метрических единиц вводятся в этот раздел учебника «Класс физики», становится все более важным организовать информацию в вашей голове. В таблице ниже перечислены некоторые количества, которые были введены до сих пор. Символ, уравнение и соответствующие метрические единицы также перечислены для каждой величины. Было бы разумно часто обращаться к этому списку или даже сделать свою собственную копию и дополнять ее по мере продвижения модуля. Некоторые учащиеся считают полезным сделать пятую колонку, в которой указано определение каждой величины.

              Количество
              Символ
              Уравнение(я)
              Стандартная метрическая единица измерения
              Прочие единицы
              Потенциальная разница

              (он же напряжение)

              ΔV
              ΔV = ΔPE/Q

              ΔV = I • R

              Вольт (В)
              Дж / С
              Текущий
              я
              I = Q/t

              I = ΔV/R

              Ампер (А)
              Ампер или C/s

              или В/Ом

              Мощность
              Р
              P = ΔPE/т

              (больше будет)

              Вт (Вт)
              Дж/с
              Сопротивление
              Р
              R = ρ • L / A

              R = ΔV / I

              Ом (Ом)
              В/А
              Энергия
              E или ΔPE
              ΔPE = ΔV • Q

              ΔPE = P • t

              Джоуль (Дж)
              В • C или

              Вт • с

              (Обратите внимание, что символ единицы измерения C представляет единицу измерения Кулоны. )

               

              В следующем разделе урока 3 мы еще раз рассмотрим количественную мощность. Новое уравнение для мощности будет введено путем объединения двух (или более) уравнений из приведенной выше таблицы.

               

               

               

              Мы хотели бы предложить …

              Зачем просто читать об этом и когда вы могли бы взаимодействовать с ним? Взаимодействие — это именно то, что вы делаете, когда используете один из интерактивов The Physics Classroom. Мы хотели бы предложить вам совместить чтение этой страницы с использованием нашего интерактивного конструктора цепей постоянного тока. Вы можете найти его в разделе Physics Interactives на нашем сайте. Конструктор цепей постоянного тока предоставляет учащимся набор для создания виртуальных схем. Легко перетащите источник напряжения, резисторы и провода на рабочую область. Соедините их, и у вас есть схема. Добавьте амперметр для измерения тока и используйте датчики напряжения для определения падения напряжения. Это так просто. И не нужно беспокоиться о поражении электрическим током (если, конечно, вы не читаете это в ванной).


              Посетите:  DC Circuit Builder


              Проверьте свое понимание

              1. Что из следующего приведет к уменьшению тока в электрической цепи? Выберите все подходящие.

              а. уменьшить напряжение

              б. уменьшить сопротивление

              в. увеличить напряжение

              д. увеличить сопротивление

               

               

              2. Некоторая электрическая цепь содержит батарею с тремя ячейками, провода и лампочку. Что из нижеперечисленного заставит лампочку светить менее ярко? Выберите все подходящие.

              а. увеличить напряжение батареи (добавить еще одну ячейку)

              б. уменьшить напряжение батареи (удалить элемент)

              в. уменьшить сопротивление цепи

              д. увеличить сопротивление цепи

               

               

               

              3. Вас, вероятно, предупредили о необходимости избегать контакта с электроприборами или даже электрическими розетками мокрыми руками. Такой контакт более опасен, когда ваши руки мокрые (а не сухие), потому что мокрые руки вызывают ____.

              а. напряжение цепи должно быть выше

              б. напряжение цепи должно быть ниже

              в. ваше сопротивление будет выше

              д. ваше сопротивление должно быть ниже

              эл. ток через вас будет ниже

               

               

               

              4. Если сопротивление цепи увеличить втрое, то ток в цепи будет ____.

              а. одна треть от

              б. в три раза больше

              с. без изменений

              д. … бред какой то! Не было бы никакой возможности сделать такой прогноз.

               

               

               

              5. Если напряжение в цепи увеличить в четыре раза, то ток в цепи будет ____.

              а. одна четвертая часть

              б. в четыре раза больше

              в. без изменений

              д. … бред какой то! Не было бы никакой возможности сделать такой прогноз.

               

               

               

              6. Цепь соединена с источником питания, резистором и амперметром (для измерения силы тока). Амперметр показывает ток 24 мА (миллиампер). Определить новый ток, если напряжение источника питания было…

              а. … увеличилось в 2 раза, а сопротивление осталось постоянным.

              б. … увеличилось в 3 раза, а сопротивление осталось постоянным.

              в. … уменьшилось в 2 раза, а сопротивление осталось постоянным.

              д. … оставался постоянным, а сопротивление увеличивалось в 2 раза.

              эл. … оставался постоянным, а сопротивление увеличивалось в 4 раза.

              ф. … оставался постоянным, а сопротивление уменьшалось в 2 раза.

              г. … увеличилось в 2 раза, а сопротивление увеличилось в 2 раза.

              час. … увеличилось в 3 раза, а сопротивление уменьшилось в 2 раза.

              я. … уменьшилось в 2 раза, а сопротивление увеличилось в 2 раза.

               

               

               

              7. Используйте уравнение закона Ома, чтобы дать численные ответы на следующие вопросы:

              а. Электрическое устройство с сопротивлением 3,0 Ом пропустит через себя ток силой 4,0 А, если на устройство будет воздействовать падение напряжения ________ Вольт.

              б. Когда на электрический нагреватель подается напряжение 120 В, через нагреватель потечет ток силой 10,0 ампер, если сопротивление составляет ________ Ом.

              в. Фонарик, который питается от 3 Вольт и использует лампочку с сопротивлением 60 Ом, будет иметь силу тока ________ Ампер.

               

               

               

              8. Используйте уравнение закона Ома, чтобы определить недостающие значения в следующих цепях.

               

               

               

              9. См. вопрос 8 выше. В цепях схем А и Б каким способом регулировали ток в цепях? А в цепях схем С и Г каким методом регулировали ток в цепях?

               

              Следующий раздел:

              Перейти к следующему уроку:

              Закон Ома: значение, формула и сопротивление

              Закон Ома был сформулирован в 1827 году немецким физиком Георгом Симоном Омом на основе экспериментов, которые он проводил с простыми электрическими цепями, содержащими провода различной длины.

              Закон Ома является одним из самых фундаментальных и важных принципов электрических цепей.

              Закон Ома утверждает, что напряжение в двух точках электрической цепи прямо пропорционально 0011 текущий , проходящий между этими двумя точками. Константа пропорциональности равна сопротивлению .

              Формула закона Ома:

              Здесь V — напряжение на проводнике, I — ток, протекающий через проводник, а R — электрическое сопротивление проводника. Сопротивление в законе Ома всегда равно постоянному значению и может быть рассчитано путем проведения серии измерений напряжения и тока в подходящем диапазоне значений перед нанесением данных на прямолинейный график и вычислением его градиента.

              Прежде чем продолжить объяснение закона Ома, мы должны рассмотреть некоторые основные понятия, касающиеся электрических цепей.

              Закон Ома: электрические цепи

              Электрическая цепь представляет собой набор электрических компонентов, соединенных проводниками, по которым может проходить электрический ток. Электрический ток состоит из движущихся электронов, которые обтекают провод под действием приложенного напряжения. Чтобы электрические компоненты могли пропускать электрический заряд, они должны быть изготовлены из электропроводящие материалы . Проводник представляет собой материал или электрический компонент, который способствует прохождению заряда (электрического тока) в одном или нескольких направлениях. Мы говорим, что такие материалы являются хорошими проводниками электричества.

              Пример электрической цепи, адаптировано из изображения: MikeRun CC BY-SA 4.0

              Металлы являются хорошими проводниками электричества, поэтому мы создаем электрические цепи, такие как те, которые используются в бытовой электронике, из меди , который обладает высокой проводимостью.

              При изучении электрических цепей мы часто различаем омических проводников и неомических проводников.

              Омический проводник — это проводник, который подчиняется закону Ома.

              График зависимости напряжения от силы тока омического проводника имеет линейную зависимость, чего нельзя сказать о неомических проводниках.

              График зависимости напряжения от тока для омического и неомического материала, Iñaki Caparros-StudySmarter Originals

              Неомический проводник не подчиняется закону Ома. Зависимость между напряжением и током для неомического проводника нелинейна.

              Поведение неомических проводников не обязательно такое же, как показано на рисунке выше. Важной особенностью является то, что существует нелинейная зависимость между током и напряжением, что означает, что график зависимости напряжения от тока не является прямой линией. Некоторыми примерами неомических проводников являются нити накала лампы или некоторые полупроводники, такие как транзисторы или диоды.

              Некоторые материалы плохо проводят электричество. Мы называем такие материалы или электрические компоненты, изготовленные из таких материалов, как изоляторы .

              Изоляторы можно использовать для замедления или остановки потока заряда, а также они могут применяться в различных реальных условиях, например, в качестве пластикового покрытия электрических проводов, которое защищает нас от поражения электрическим током.

              Закон Ома: напряжение

              Напряжение также известно как разность потенциалов. Разность потенциалов между двумя точками проводника равна разнице электрических потенциалов между двумя точками. Разность потенциалов в электрической цепи создается элементами или батареями. В стандартных единицах мы выражаем разность потенциалов/напряжение в вольт (В).

              В электрических цепях напряжение генерируется ячейкой или батареей, у которых есть положительная клемма с более высоким потенциалом и отрицательная клемма с более низким потенциалом.

              Закон Ома: Ток

              Ток — это скорость потока электрического заряда. Прибор, который мы используем для измерения силы тока в электрической цепи, называется амперметром . Стандартной единицей электрического тока является А мпер (А).

              Закон Ома: сопротивление

              Сопротивление — это мера сопротивления проводников потоку электричества. Стандартной единицей измерения сопротивления является O hm (Ом). Сопротивление электрического проводника увеличивается с длиной и уменьшается с толщиной. Сопротивление также зависит от типа материала, из которого он изготовлен.

              Алюминиевый провод имеет более высокое сопротивление, чем медный провод той же длины и площади поперечного сечения, что означает, что медь является лучшим проводником электричества, чем алюминий. Но алюминиевый провод имеет меньшее сопротивление, чем медный провод, в четыре раза превышающий его длину.

              Чтобы увеличить электрическое сопротивление электрической цепи, мы можем добавить компонент, называемый резистором . Фиксированный резистор подчиняется закону Ома, и разные фиксированные резисторы имеют разные постоянные сопротивления для разных целей.

              Примеры резисторов с различным сопротивлением в электрических цепях, wikipedia.org

              Вывод закона Ома

              Точного вывода формулы закона Ома не существует. Как упоминалось ранее, закон был сформулирован в 1827 году Георгом Симоном Омом, но это эмпирический закон, а это означает, что он изначально был основан на наблюдениях, а не выведен из первых принципов. Ом открыл закон, наблюдая за поведением омических проводников при подаче на них тока. Основываясь на полученных данных, Ом констатировал наличие линейной зависимости между током и силой тока, но теоретически закон не вывел.

              Новый взгляд на формулу закона Ома

              Теперь, когда мы выяснили, что означает каждый термин в законе Ома, давайте вспомним формулу: уравнение, что существует прямая зависимость между напряжением и силой тока. Мы говорим, что напряжение на проводнике прямо пропорционально протекающему по нему току. Это означает, что если мы увеличим ток, проходящий через проводник, в определенный раз, напряжение увеличится во столько же раз, и наоборот. Например, если мы удвоим разность потенциалов на проводе, ток, проходящий через него, также удвоится.

              Мы можем изменить формулу закона Ома, чтобы сделать сопротивление или ток предметом формулы. Если мы знаем значения любых двух переменных в законе Ома, мы можем вычислить значение оставшейся отсутствующей переменной. Например, если мы знаем ток, проходящий через компонент в электрической цепи, и мы также знаем разность потенциалов между двумя сторонами компонента, мы можем рассчитать сопротивление проводника по следующей формуле:

              Это по-прежнему закон Ома, только измененный, чтобы сделать R предметом формулы. Точно так же, если мы знаем разность потенциалов на компоненте и его сопротивление, мы можем вычислить ток I, проходящий через него: мы знаем значения двух других.

              Треугольник закона Ома с напряжением (V), током (I) и сопротивлением (R), Iñáki Caparros-StudySmarter Originals

              Этот треугольник называется треугольником закона Ома . Чтобы напомнить себе, как вычислять каждый из V, I и R, мы создали треугольник с V в верхнем сегменте и I и R в нижних сегментах. Чтобы вычислить значение любой из переменных в нижней части треугольника, мы просто делим значение V на значение другой оставшейся переменной в нижней части треугольника. V просто рассчитывается путем умножения значений двух переменных в нижнем сегменте треугольника, а именно I и R.

              Если вам трудно переставить уравнения, вы можете предпочесть вычислить V, I и R, используя треугольник закона Ома. Только не забудьте нарисовать треугольник и разделить его на три части: верхняя часть содержит V, а две нижние части содержат I и R.

              Батарея напряжением 9 В производит ток силой 3 А в проводе. Каково сопротивление провода?

              Для решения этой задачи воспользуемся законом Ома. Используя наши знания о преобразовании уравнений или законе треугольника Ома, мы находим, что формула для вычисления сопротивления:

              Чтобы найти одну из двух переменных нижнего уровня треугольника закона Ома, мы делим напряжение на другую переменную нижнего уровня. В этом случае делим V на I, чтобы найти R.

              Следовательно, сопротивление провода в данном примере равно:

              Определить напряжение электрической цепи с током 0,5 А и сопротивлением 20 Ом .

              В этом случае нам нужно использовать первую форму закона Ома. Формулу закона Ома для напряжения находим по треугольнику закона Ома:

              Теперь мы можем ввести данные для электрической цепи, приведенные в вопросе, подставив 0,5 А для тока и 20 Ом для сопротивления: переключатель замкнут.

              Электрическая цепь с напряжением V= 30 В и сопротивлением R= 10 Ом, Iñaki Caparros-StudySmarter Originals

              Как видно из рисунка выше, электрическая цепь имеет разность потенциалов 30 В на своих клеммах, и резистор имеет сопротивление 10 Ом. Мы должны изменить первоначальную форму формулы закона Ома. Если мы посмотрим на треугольник закона Ома, мы можем изменить формулу для расчета тока:

              Используя значения переменных, представленных на рисунке, мы можем рассчитать ток следующим образом:

              Закон Ома – основные выводы в двух точках электрической цепи пропорционально

              току , проходящему через нее, а константой пропорциональности является сопротивление .
            • Общее выражение для закона Ома равно В = ИК .
            • Электрическая цепь представляет собой набор электрических компонентов, соединенных проводами, по которым протекает электрический заряд. Он состоит из элементов из проводящих материалов, которые позволяют электрическим зарядам двигаться по ним. Материалы, плохо проводящие электричество, называются изоляторами .
            • Напряжение — это разность электрических потенциалов между двумя точками. Стандартной единицей напряжения является вольт (В) .
            • Ток — скорость потока электрического заряда. Стандартной единицей тока является ампер (А) .
            • Сопротивление — это свойство проводника сопротивляться потоку электрического заряда. Стандартной единицей сопротивления является Ом (Ом) .
            • Закон Ома можно использовать для расчета напряжения, сопротивления и тока электрических проводников при условии, что известны две другие переменные. Один из способов запомнить формулу закона Ома — это Треугольник закона Ома .

            Закон Ома – Связь между напряжением, током и сопротивлением – Производство печатных плат и сборка печатных плат

            Основы электронной техники основаны на принципе закона Ома. Без закона Ома ток не может течь по проводам, не будет напряжения и не будет работать ни одна цепь в мире. Это такой же важный закон электроники, как и закон Ньютона в физике. Однако есть много других важных принципов и законов электроники, которые необходимо выделить, но в этой статье мы изучим основы закона Ома и три его важных параметра: напряжение (В), ток (I) и сопротивление (R).

            Электронная схема — это не что иное, как замкнутый контур проводящих проводов и электронных компонентов, соединенных друг с другом.

            Напряжение (В):

            Напряжение можно описать как разность потенциалов между двумя точками цепи. Его стандартная международная единица измерения (SI)  Вольт . Напряжение — это движущая сила , приводящая в движение электрический заряд в цепи. Чем больше разность потенциалов, тем больше поток заряда (ток) между двумя точками. Напряжение измеряется «поперек» или «между» двумя точками. В некоторых учебниках напряжение обозначается как «Электродвижущая сила (ЭДС)». Поэтому обозначается буквой «Е». Он называется электродвижущим, потому что мотивирует поток электронов в цепи или проводнике. Мы можем использовать либо V, либо E для обозначения, но оба варианта правильны.

            Запросить производство и сборку печатных плат

            Напряжение = Джоули/Кулоны:

            Другое определение напряжения можно объяснить с точки зрения энергии (Джоули) и заряда (Кулоны). Напряжение или разность потенциалов (P.D) можно определить как количество энергии, затраченное на 1 кулон заряда, чтобы перевести его из одной точки цепи в другую точку цепи.

            Джоуль — единица измерения энергии в системе СИ

            Ток (I):

            Ток можно определить как количество заряда, протекающего из одной точки цепи в другую в единицу времени.  Международная стандартная единица (СИ)  Ампер . Течение тока можно рассматривать как аналог течения воды в трубе. Ток измеряется «через» резистор или любой другой электронный компонент. Ток обозначается буквой «I», которая происходит от слова «интенсивность» или сила тока.

            Заряд: . Заряд может быть положительным или отрицательным зарядом в зависимости от Направление из потока Electrons . Обычный поток заряда можно определить как заряд, выходящий из положительной клеммы аккумулятора и входящий в отрицательную клемму. Однако в концепции электронного потока электроны покидают клемму с отрицательным зарядом и входят в клемму с положительным зарядом. В обоих случаях ток будет течь, потому что этот процесс происходит очень быстро и непрерывно. Единицей заряда в СИ является «Кулон (Кл)». Один электронный заряд = Заряд электрона = 1,6×10 -19 С .

            Один кулон (Кл) = 1 Кл заряд = 6,24×10 18 электронов, протекающих через определенную точку цепи за 1 секунду.

            Сопротивление (R):

            Сопротивление — это сопротивление, создаваемое в потоке тока электронным компонентом, называемым «резистор».  Сопротивление может создаваться другими электронными компонентами, кроме резистора. Сопротивление на самом деле является препятствием или помехой, создаваемой цепью для ограничения протекания тока. Сопротивление измеряется «поперек» любых двух точек или «между» двумя клеммами в цепи. Его единицей измерения Standard International (S.I) является «Ом Ω». Популярными единицами сопротивления являются Ом, кОм, МОм.

            Запросить производство и сборку печатных плат сейчас

            Связь между V, I и R:

            Математическая связь между этими тремя величинами является линейной. Это означает, что из 3, если мы знаем 2 величины, мы можем найти третью. В основном сопротивление является унаследованным свойством любого проводника, которое почти считается постоянным. Например, кусок провода с площадью поперечного сечения «А» и длиной «L», а его удельное сопротивление равно ρ. Сопротивление можно рассчитать как

            Это показывает, что сопротивление (R) проводника/провода зависит от его длины (L) и калибра (A). Где ρ — константа (удельное сопротивление) для материала проводника, такого как медь или серебро.

            Следовательно, теперь мы можем сказать, что напряжение и ток зависят друг от друга, принимая во внимание постоянное сопротивление R.

            Закон Ома:

            Закон Ома гласит, что величина тока (I), протекающего через любой проводник, прямо пропорциональна разности потенциалов (напряжению V) на нем при данной температуре при условии сопротивления (R) остается постоянным.

            Пример-1:

            Мы рассмотрим базовый пример того, что такое ток, напряжение и сопротивление. На приведенной ниже схеме мы видим, что батарея постоянного тока напрямую подключена к лампе через проводящие провода. Направление тока «условное (от + к -)».

            Здесь мы можем создать три сценария. 1- В и Р известны, я неизвестен. 2- V и I знают R неизвестно 3- I и R известны, но V неизвестно.

            Сценарий-1:

            Предположим, что напряжение батареи составляет В = 12 В постоянного тока по показаниям «вольтметра», а лампа представляет собой нагрузку с сопротивлением = 4 Ом, значение которого указано в спецификации лампы. Теперь рассчитайте величину тока, протекающего в цепи

            Запросите предложение по изготовлению и сборке печатных плат сейчас

            Решение:

            « нагрузка» 9025 устройство или электронное оборудование который вытягивает ток из цепи и, следовательно, падает напряжение на себя и падает последовательно.

            Применяя закон Ома, мы получаем

            Сценарий-2 :

            Допустим, напряжение батареи составляет 12 В постоянного тока, а ток в цепи, измеренный амперметром, составляет 6 ампер. Узнайте сопротивление нагрузки.

            Решение:

            Ток в цепи можно измерить, подключив амперметр последовательно .

            Теперь снова применим закон Ома и получим результат.

            Сценарий-3:

            Теперь у нас есть сопротивление лампы, уже известное из паспорта производителя. R = 10 Ом, а ток измеряется амперметром и составляет 1,2 ампера. Рассчитать напряжение батареи.

            Запросить производство и сборку печатных плат сейчас

            Решение: применить закон Ома.

            До сих пор мы анализировали очень базовое понимание цепей постоянного тока. Где напряжение и ток были постоянными и стабильными в течение определенного периода времени. Однако цепи переменного тока не являются стабильными в течение определенного периода времени. Напряжение переменного тока и ток переменного тока изменяются с течением времени.

            Заключение:

            Закон Ома является основой решения любой электронной или электротехнической проблемы. Либо это маломощная электронная схема с низким напряжением, либо цепь постоянного тока, небольшие миниатюрные схемы на основе микроконтроллера, аналоговые или цифровые схемы, микровольтные или милливольтовые схемы, либо это анализ цепи большой мощности или большего напряжения, такой как выпрямители, мосты. , инверторы или любые другие электронные схемы переменного или постоянного тока, все они полностью зависят от закона Ома. Без запоминания этого фундаментального принципа электроники вы не сможете решить ни одну проблему в области электроники и электротехники.

            Закон Ома, сила тока, напряжение и сопротивление

            Закон Ома

            Закон Ома описывает основные математические соотношения электричества. Закон был назван в честь немецкого физика Джорджа Симона Ома (1789–1854). По сути, закон Ома гласит, что ток (поток электронов) через проводник прямо пропорционален напряжению (электрическому давлению), приложенному к этому проводнику, и обратно пропорционален сопротивлению проводника. Единица, используемая для измерения сопротивления, называется ом.

            Ом обозначается греческой буквой омега (Ω). В математических формулах заглавная буква R обозначает сопротивление. Сопротивление проводника и приложенное к нему напряжение определяют количество ампер тока, протекающего по проводнику. Таким образом, сопротивление 1 Ом ограничивает протекание тока до 1 ампера в проводнике, к которому приложено напряжение 1 вольт. Основная формула, полученная из закона Ома: E = I × R (E = электродвижущая сила, измеренная в вольтах, I = протекающий ток, измеренный в амперах, и R = сопротивление, измеренное в омах). Эту формулу также можно записать для определения тока или сопротивления:

            I = E
            R
            R = E
            I


            Рисунок 1. Закон ом использовался для рассчитания, чтобы рассчитывать, как будет проходить. Источник питания постоянного тока

            Закон Ома представляет собой основу математических формул, которые предсказывают реакцию электричества на определенные условия. [Рисунок 1] Например, закон Ома можно использовать для расчета того, что лампа с сопротивлением 12 Ом (Ом) пропускает ток силой 2 ампера при подключении к источнику питания постоянного тока напряжением 24 В.

            Пример 1
            28-вольтовая цепь посадочных огней имеет лампу с сопротивлением 4 Ом. Вычислите полный ток цепи.

            I = E
            R
            I = 28 Вольт

            I = 7 Amps

            Пример 2
            Схема загрузки 28-вольта имеет ток 6,5 амперс. Каликуляция сопротивления Deice Boot.

            R =   E 
                    I

            R =  28 вольт

                    6,5 ампер
            R = 4,31 Ом

            Пример 3
            Такси-фонарь имеет сопротивление 4,9 Ом и общий ток 2,85 ампер. Рассчитайте напряжение в системе.

            E = I x R
            E = 2,85 x 4,9 Ом
            E — 14 В

            При поиске и устранении неисправностей электрических цепей самолета всегда полезно учитывать закон Ома. Хорошее понимание взаимосвязи между сопротивлением и протеканием тока может помочь определить, есть ли в цепи обрыв или короткое замыкание. Память о том, что низкое сопротивление означает повышенный ток, может помочь объяснить, почему выключаются автоматические выключатели или перегорают предохранители. Почти во всех случаях нагрузки самолета подключаются параллельно друг другу; поэтому на все нагрузки подается постоянное напряжение, а ток, протекающий через нагрузку, зависит от сопротивления этой нагрузки.

            На рис. 2 показано несколько способов использования закона Ома для расчета тока, напряжения и сопротивления.

            Рис. 2. Схема закона Ома Это движение электронов называется током, потоком или током. С практической точки зрения это движение электронов должно происходить внутри проводника (провода). Ток обычно измеряется в амперах. Символ тока — I, а символ ампер — A.

            Течение тока на самом деле представляет собой движение свободных электронов внутри проводников. Обычные проводники включают медь, серебро, алюминий и золото. Термин «свободный электрон» описывает состояние некоторых атомов, при котором внешние электроны слабо связаны со своим родительским атомом. Эти слабо связанные электроны легко побуждаются двигаться в заданном направлении, когда в цепь подается внешний источник, например батарея. Эти электроны притягиваются к положительной клемме батареи, а отрицательная клемма является источником электронов. Таким образом, мерой силы тока на самом деле является количество электронов, проходящих через проводник за заданный промежуток времени.

            Общепринятой единицей силы тока является ампер (А). Один ампер (А) тока эквивалентен 1 кулону (Кл) заряда, проходящего через проводник за 1 секунду. Один кулон заряда равен 6,28 × 1018 электронов. Очевидно, что термин «ампер» гораздо удобнее, чем кулон. Единица кулонов просто слишком мала, чтобы быть практичной.

            Когда ток течет в одном направлении, это называется постоянным током (DC). Далее в тексте обсуждается форма тока, который периодически колеблется взад и вперед внутри цепи. Настоящее обсуждение касается только использования постоянного тока. Следует отметить, что, как и в случае с движением любой массы, движение электронов (поток тока) происходит только тогда, когда присутствует сила, толкающая электроны. Эту силу обычно называют напряжением (более подробно она описана в следующем разделе). Когда к проводнику приложено напряжение, электродвижущая сила создает электрическое поле внутри проводника, и возникает ток. Электроны не движутся в прямом направлении, а подвергаются повторяющимся столкновениям с другими соседними атомами внутри проводника. Эти столкновения обычно выбивают другие свободные электроны из их атомов, и эти электроны движутся к положительному концу проводника со средней скоростью, называемой скоростью дрейфа, которая является относительно низкой скоростью. Чтобы понять почти мгновенную скорость воздействия тока, полезно представить себе длинную трубу, наполненную стальными шариками. [Рисунок 3]

            противоположный конец трубы. Таким образом, электрический ток можно рассматривать как мгновенный, хотя он и является результатом относительно медленного дрейфа электронов.


            Традиционная теория тока и электронная теория

            Есть две конкурирующие школы мысли относительно потока электричества. Двумя объяснениями являются обычная текущая теория и электронная теория. Обе теории описывают движение электронов в проводнике. Они просто объясняют направление движения тока. Обычно при устранении неполадок или подключении электрических цепей можно применять любую теорию, если она используется последовательно.

            Обычная теория тока была первоначально выдвинута Бенджамином Франклином, который рассуждал о том, что ток течет из положительного источника в отрицательный источник или в область, в которой отсутствует избыток заряда. Обозначение, присвоенное электрическим зарядам, было положительным (+) для избытка заряда и отрицательным (-) для отсутствия заряда. Тогда казалось естественным визуализировать поток тока как от положительного (+) к отрицательному (-). Позднее были сделаны открытия, доказавшие, что верно как раз обратное. Электронная теория описывает, что на самом деле происходит в случае обилия электронов, вытекающих из отрицательного (-) источника в область, где электронов не хватает, или в положительный (+) источник. Как обычный поток, так и электронный поток используются в промышленности.


            Электродвижущая сила (напряжение)

            Напряжение проще всего описать как силу электрического давления. Это электродвижущая сила (ЭДС), или толчок или давление от одного конца проводника к другому, который в конечном итоге перемещает электроны. Символом ЭДС является заглавная буква E. ЭДС всегда измеряется между двумя точками, а напряжение считается значением между двумя точками. Например, на клеммах типичной аккумуляторной батареи самолета напряжение может быть измерено как разность потенциалов 12 вольт или 24 вольта. То есть между двумя клеммами батареи есть напряжение, необходимое для проталкивания тока по цепи. Свободные электроны на отрицательном полюсе батареи движутся к избыточному количеству положительных зарядов на положительном полюсе. Конечным результатом является поток или ток через проводник. В проводнике не может быть потока, если к нему не приложено напряжение от батареи, генератора или наземной силовой установки. Разность потенциалов, или напряжение в любых двух точках электрической системы, можно определить с помощью:

            V1 – V2 = VDrop

            Пример
            Напряжение в одной точке составляет 14 вольт. Напряжение во второй точке цепи равно 12,1 вольта. Чтобы рассчитать падение напряжения, используйте приведенную выше формулу, чтобы получить общее падение напряжения 1,9 вольт.

            Рисунок 4 иллюстрирует поток электронов электрического тока. Два соединенных между собой резервуара для воды демонстрируют, что, когда между двумя резервуарами существует разница давлений, вода течет до тех пор, пока оба резервуара не выровняются. На рис. 4 показано, что уровень воды в резервуаре А находится на более высоком уровне, показывающем 10 фунтов на квадратный дюйм (psi) (более высокая потенциальная энергия), чем уровень воды в резервуаре B, показывающем 2 фунта на квадратный дюйм (более низкая потенциальная энергия). Между двумя резервуарами существует разность потенциалов 8 фунтов на квадратный дюйм. Если клапан на соединительной линии между резервуарами открыт, вода перетекает из резервуара А в резервуар В до тех пор, пока уровень воды (потенциальная энергия) в обоих резервуарах не сравняется. Важно отметить, что не давление в резервуаре А вызвало течение воды; скорее, это была разница в давлении между резервуарами A и резервуарами B, которая вызвала поток. Это сравнение иллюстрирует принцип, согласно которому электроны движутся, когда есть путь, из точки с избытком электронов (более высокая потенциальная энергия) в точку с недостатком электронов (более низкая потенциальная энергия). Сила, вызывающая это движение, представляет собой разность потенциалов электрической энергии между двумя точками. Эта сила называется электрическим давлением (напряжением), разностью потенциалов или электродвижущей силой (силой движения электронов). 9Рис. 4. Разность давлений В любой электрической цепи при приложении к ней напряжения возникает ток. Сопротивление проводника определяет величину тока, протекающего при данном напряжении. Как правило, чем больше сопротивление цепи, тем меньше ток. Если сопротивление уменьшить, то ток увеличится. Эта зависимость носит линейный характер и известна как закон Ома. Например, если сопротивление цепи удваивается, а напряжение поддерживается постоянным, то ток через резистор уменьшается вдвое.

            Нет четкой границы между проводниками и изоляторами; при надлежащих условиях все типы материалов проводят некоторый ток. Материалы, обладающие сопротивлением протеканию тока посередине между лучшими проводниками и самыми плохими проводниками (изоляторами), иногда называют полупроводниками и находят наибольшее применение в области транзисторов.

            Лучшими проводниками являются материалы, главным образом металлы, которые обладают большим количеством свободных электронов. И наоборот, изоляторы — это материалы с небольшим количеством свободных электронов. Лучшими проводниками являются серебро, медь, золото и алюминий, но в качестве проводников можно использовать и некоторые неметаллы, например углерод и воду. Такие материалы, как резина, стекло, керамика и пластик, являются настолько плохими проводниками, что их обычно используют в качестве изоляторов. Течение тока в некоторых из этих материалов настолько мало, что его обычно считают нулевым.

            Факторы, влияющие на сопротивление

            Сопротивление металлического проводника зависит от типа материала проводника. Было указано, что некоторые металлы обычно используются в качестве проводников из-за большого количества свободных электронов на их внешних орбитах. Медь обычно считается лучшим доступным материалом для проводников, поскольку медная проволока определенного диаметра оказывает меньшее сопротивление току, чем алюминиевая проволока того же диаметра. Однако алюминий намного легче меди, и по этой причине, а также из соображений стоимости алюминий часто используется, когда важен фактор веса.

            Рисунок 5. Сопротивление зависит от длины проводника

            Сопротивление металлического проводника прямо пропорционально его длине. Чем больше длина провода данного сечения, тем больше сопротивление. На рис. 5 показаны две проволочные жилы разной длины. Если к двум концам проводника длиной 1 фут приложено электрическое давление 1 вольт, а сопротивление движению свободных электронов принимается равным 1 Ом, то ток ограничивается 1 ампер. Если проводник того же размера удвоить в длину, те же самые электроны, приведенные в движение приложенным 1 вольтом, теперь обнаруживают удвоенное сопротивление.


            RELATED POSTS

            • Aircraft Electrical System
            • Electromagnetic Generation of Power
            • Alternating Current (AC) Introduction
            • Opposition to Current Flow of AC
            • Aircraft Batteries
            • Wire Identification
            • Wire Installation and Routing and Пучки проводов для шнуровки и связывания
            • Концевая заделка проводов

            Предыдущий пост Следующий пост

            Что такое закон Ома? (Простое объяснение)

            Содержание

            Что такое закон Ома?

            Закон Ома гласит, что электрический ток, протекающий через любой проводник, прямо пропорционален разности потенциалов (напряжению) между его концами, при условии, что физические условия проводника не меняются.

            Другими словами, отношение разности потенциалов между любыми двумя точками проводника к току, протекающему между ними, является постоянным при условии, что физические условия (например, температура и т. д.) не меняются.

            Математически закон Ома можно выразить следующим образом: (),

          • I — ток через проводник в Амперах (А),
          • В — напряжение или разность потенциалов, измеренная на проводнике в Вольтах (В).
          • Закон Ома применим как к постоянному, так и к переменному току.

            Зависимость между разностью потенциалов или напряжением (V), током (I) и сопротивлением (R) в электрической цепи впервые была обнаружена немецким физиком Джорджем Симоном Омом.

            Единица сопротивления Ом () была названа в честь Джорджа Саймона Ома.

            Как работает закон Ома?

            Согласно определению закона Ома, ток, протекающий по проводнику или резистору между двумя точками, прямо пропорционален разности напряжений (или разности потенциалов) на проводнике или резисторе.

            Но… это может быть немного сложно понять.

            Итак, давайте лучше интуитивно поймем закон Ома, используя некоторые аналогии.

            Аналогия 1

            Рассмотрим резервуар для воды, расположенный на определенной высоте над землей. В нижней части резервуара для воды находится шланг, как показано на рисунке ниже.

            Аналогия 1
            • Давление воды в паскалях на конце шланга аналогично напряжению или разности потенциалов в электрической цепи.
            • Расход воды в литрах в секунду аналогичен электрическому току в кулонах в секунду в электрической цепи.
            • Ограничители расхода воды, такие как отверстия, расположенные в трубах между двумя точками, являются аналогами резисторов в электрической цепи.

            Таким образом, расход воды через ограничитель отверстия пропорционален перепаду давления воды на ограничителе.

            Аналогично, в электрической цепи ток, протекающий по проводнику или резистору между двумя точками, прямо пропорционален разности напряжений или разности потенциалов на проводнике или резисторе.

            Можно также сказать, что сопротивление потоку воды зависит от длины трубы, материала трубы и высоты резервуара, расположенного над землей.

            Ом работает аналогичным образом в электрической цепи, что электрическое сопротивление, оказываемое протеканию тока, зависит от длины проводника и материала используемого проводника.

            Аналогия 2

            Простая аналогия между гидравлическим водяным контуром и электрическим контуром для описания работы закона Ома показана на изображении ниже.

            Аналогия 2

            Как показано, если давление воды постоянно, а ограничение увеличивается (что затрудняет течение воды), то скорость потока воды уменьшается.

            Аналогичным образом, в электрической цепи, если напряжение или разность потенциалов постоянны, а сопротивление увеличивается (что затрудняет протекание тока), скорость протекания электрического заряда, т. е. сила тока, уменьшается.

            Теперь, если ограничение потока воды постоянно, а давление насоса увеличивается, скорость потока воды увеличивается.

            Аналогичным образом, в электрической цепи, если сопротивление постоянно, а разность потенциалов или напряжение увеличивается, скорость протекания электрического заряда, т. е. ток, увеличивается.

            Формула закона Ома

            Связь между напряжением или разностью потенциалов, током и сопротивлением может быть записана тремя различными способами.

            Если мы знаем любые два значения, мы можем вычислить третье неизвестное значение, используя соотношение закона Ома. Таким образом, закон Ома очень полезен в электронике и электрических формулах и расчетах.

            Если известный электрический ток протекает через известное сопротивление, падение напряжения на сопротивлении можно рассчитать по соотношению

               

            Если известное напряжение приложено к неизвестному сопротивлению и ток, протекающий через сопротивление, также известен, то значение неизвестного сопротивления можно рассчитать по соотношению

               

            Формула мощности по закону Ома

            Передаваемая мощность является произведением напряжения питания и электрического тока.

            (1)  

            Теперь, подставив в уравнение (1), мы получим

            (2)  

            Эта формула известна как формула омических потерь или формула резистивного нагрева.

            Теперь, подставив в уравнение (1), мы получим

            (3)

            Из приведенного выше соотношения мы можем определить мощность, рассеиваемую в сопротивлении, если известны либо напряжение и сопротивление, либо ток и сопротивление.

            Мы также можем определить неизвестное значение сопротивления, используя приведенное выше соотношение, если известно напряжение или ток.

               

            Если известны любые две переменные мощности, напряжения, тока и сопротивления, то с помощью закона Ома мы можем определить две другие переменные.

               

               

            Ограничения закона Ома

            Некоторые ограничения закона Ома обсуждаются ниже.

            • Закон Ома не распространяется на все неметаллические проводники. Например, для карбида кремния зависимость задается формулой где K и m — константы, а m<1.
            • Закон Ома не применим к следующим нелинейным элементам.
            1. Resistance
            2. Capacitance
            3. Semiconductors
            4. Vacuum tubes
            5. Electrolytes
            6. Carbon resistors  
            7. Arc Lamps
            8. Zener diode

            (Note that Non-linear elements are those in which the relation between current and voltage является нелинейным, т. е. ток не совсем пропорционален приложенному напряжению.)

            • Закон Ома применим к металлическим проводникам только при постоянной температуре. Если температура меняется, закон не применяется.
            • Закон Ома также неприменим к односторонним сетям. Обратите внимание, что односторонняя сеть содержит односторонние элементы, такие как транзисторы, диоды и т. д. Односторонние элементы — это те элементы, которые пропускают ток только в одном направлении.

            Треугольник закона Ома

            Основные формулы закона Ома приведены под треугольником закона Ома.

            Треугольник закона Ома

            Круговая диаграмма закона Ома

            Основные формулы закона Ома представлены на приведенной ниже круговой диаграмме закона Ома.

            Круговая диаграмма закона Ома

            Практические задачи закона Ома

            Пример 1

            Как показано на схеме ниже, ток силой 4 А протекает через сопротивление 15 Ом. Определить падение напряжения в цепи по закону Ома.

            Решение:

            Данные: и

            Согласно закону Ома,

               

            Таким образом, используя уравнение закона Ома, мы получаем падение напряжения в цепи 60 В. . Определить силу тока, протекающего через резистор, по закону Ома.

            Решение:

            Данные: и

            Согласно закону Ома,

               

            Таким образом, используя уравнение закона Ома, мы получаем, что ток, протекающий через резистор, равен 2 А.

            Пример 3

            Как показано на схеме ниже, напряжение питания составляет 24 В, а ток, протекающий через неизвестное сопротивление, равен 2 А. Определите неизвестное значение сопротивления, используя закон Ома.

            Решение:

            Данные: и

            Согласно закону Ома,

               

            Таким образом, используя уравнение закона Ома, мы получаем неизвестное значение сопротивления .

            Применение закона Ома

            Некоторые приложения закона Ома включают:

            • Для расчета неизвестной разности потенциалов или напряжения, сопротивления и протекания тока в электрической цепи.
            • Закон Ома используется в электронной схеме для определения внутреннего падения напряжения на электронных компонентах.
            • Закон Ома используется в измерительных цепях постоянного тока, в частности, в амперметрах постоянного тока, в которых для отвода тока используется низкоомный шунт.

            Закон Ома

            Закон Ома закон является основным правилом электричества объясняет отношения между электрическим током, напряжением и сопротивлением. Ом Закон был назван в честь немецкого физика Георга Ома, который открыл это правило или закон.

            Ом заявление закона

            Ом Закон гласит, что электрический ток, протекающий через проводник прямо пропорционален напряжению и обратно пропорциональна сопротивлению. В других Другими словами, электрический ток, протекающий по проводнику увеличивается с увеличением напряжения (если сопротивление не изменилось), в то время как электрический ток, протекающий через проводник уменьшается с увеличением сопротивления проводника (Если напряжение не изменилось).

            Ом закон математически записывается как

            Где V = Напряжение, приложенное к проводнику,
            I = Электрический ток, протекающий через кондуктор,
            R = сопротивление проводника

            Электрический определение тока, напряжения и сопротивления

            Электрический ток: числа бесплатно электроны проходят через проводник за одну секунду называется электрическим током. Электрический ток измеряется в Ампер (А).

            Напряжение: Разница в электрическом потенциальная энергия заряженных частиц между двумя точки внутри эл. поле называется напряжением. Свободные электроны на более высокий потенциал имеет больше электрической потенциальной энергии тогда как свободные электроны при более низком потенциале имеют меньше электрическая потенциальная энергия. Напряжение измеряется в вольтах (В).

            Сопротивление: Сопротивление — это сила, противодействующая потоку электронов. электроны, свободно движущиеся по проводнику, будут постоянно сталкиваются с атомами (которые действуют как барьеры). Это приводит к тому, что свободные электроны теряют свою энергию. Следовательно, электрический ток уменьшается. Сопротивление измеряется в омах (Ом).

            Ом объяснение закона с примером

            понятие закона Ома легко понять с водой аналогия. Разница в давлении воды между двумя точками в баке вода течет. Здесь разница в давлении воды сравнивается с напряжения или разности потенциалов. Скорость, с которой течет вода в секунду по сравнению с электрическим током. Препятствие что уменьшает расход воды по сравнению с сопротивлением.

            Подано напряжение проводнику


            Когда к проводнику приложено напряжение, свободные электроны кинетический энергии и начинает течь от более высокого потенциала проводника к более низкому потенциалу проводника. В течение путешествие из одного конца в другой конец проводника свободные электроны сталкиваются с атомами или ионами.

            Когда свободные электроны, движущиеся в проводнике, сталкиваются с атомы теряют свою кинетическую энергию. Энергетическая потеря свободного электроны выделяются в виде тепла. Однако из-за непрерывная подача внешнего электрического поля или напряжения, свободные электроны снова ускоряются. Свободные электроны движутся через проводник снова сталкиваются с атомами и теряют свою кинетическая энергия.

            В Таким образом, свободные электроны, движущиеся через проводник постоянно сталкиваются с атомами. Из-за этого непрерывного столкновения средняя скорость свободных электронов уменьшается. Следовательно, количество свободных электронов, перетекающих из одной точки в другая точка внутри проводника уменьшается. Следовательно, электрический ток уменьшается.

            Если напряжение применительно к проводнику увеличивается


            Если напряжение или электрическое поле, приложенное к проводнику, равно увеличивается, свободные электроны получают большое количество кинетической энергия. Следовательно, скорость свободных электронов увеличивается.

            Когда свободные электроны, движущиеся по проводнику, сталкиваются с атомами они теряют свою кинетическую энергию. Однако из-за к непрерывной подаче напряжения свободные электроны снова достичь их скорости. Свободные электроны, движущиеся через проводник снова сталкивается с атомами и теряет их кинетическая энергия.

            В Таким образом, свободные электроны, протекающие через проводник постоянно сталкиваются с атомами. Из-за этого непрерывное столкновение, средняя скорость свободного электронов уменьшается. Однако мы предоставили больше энергии или электрического поля к свободным электронам, чем в предыдущем случае. Следовательно, средняя скорость свободных электронов увеличивается если сравнивать с предыдущим случаем. Следовательно, электрическая ток увеличивается.

            Если напряжение прикладываемое к проводнику, остается постоянным, а сопротивление равно увеличилось


            Если напряжение или электрическое поле, приложенное к свободным электронам в проводник остается постоянным, а сопротивление увеличивается. проводник, число столкновений с атомами увеличивается. В результате средний дрейф скорость свободных электронов уменьшается. Следовательно электрический ток уменьшается.

            сила притяжения ядра также влияет на скорость свободные электроны. Сила притяжения ядра всегда пытается притянуть свободные электроны. Это уменьшает скорость свободных электронов. В результате электрический ток уменьшается.

            Разное формулировка уравнения закона Ома

            In электрические цепи, уравнение закона Ома состоит из трех разными способами в зависимости от того, что мы хотим решить.

            Ом уравнение закона для нахождения электрического тока:

            In чтобы решить электрический ток, уравнение закона Ома записывается как

            приведенное выше уравнение говорит нам, что электрический ток увеличивается при увеличении напряжения (если сопротивление не изменилось) тогда как электрический ток уменьшается с увеличением сопротивление (если напряжение не меняется).

            Ом уравнение закона для нахождения сопротивления:

            В чтобы решить сопротивление, уравнение закона Ома записывается как

            приведенное выше уравнение говорит нам, что сопротивление увеличивается с увеличение напряжения (если электрический ток не изменяется) тогда как сопротивление уменьшается с увеличением электрического ток (если напряжение не меняется).

            Ом Уравнение закона для нахождения напряжения:

            In Чтобы решить напряжение, уравнение закона Ома записывается как

            приведенное выше уравнение говорит нам, что напряжение увеличивается с увеличение электрического тока (если сопротивление не изменилось) или напряжение увеличивается с увеличением сопротивления (если ток не меняется).

            Добавить комментарий

            Ваш адрес email не будет опубликован. Обязательные поля помечены *